You are on page 1of 80

Linear Functions,

Equations,
Inequalities and Its
Applications
 Learning Outcomes
On completion of this unit, students will be able to:
 Define functions, inverse and composite functions.
 Draw the graphs of linear functions.
 Find the slope and intercepts of a line.
 Solve linear inequalities and use their properties.
 Define absolute values and use their properties (without theoretical proof).
 Apply the linear functions in economics and management.
 Use excel for linear functions, equations, inequalities and its Applications.

Linear Functions, Equations And Inequalities


1.1 Introduction
Functions are used in different areas of mathematics and they are the major tools for describing the real
world in mathematical terms. Using the objects, we can make a rule of correspondence that associates
each elements of one set with elements of another set. In mathematics, a single valued correspondence is
known as a function. When one quantity depends on another, then we associate them by means of a
function. For example, the demand for a commodity depends upon its price, the interest paid on an
amount invested depends on the length of time the money is invested. In each cases, one quantity depends
on another, these arise the notion of functions. Before defining function, we give the definition of variable
and constant.
Constant and Variable
Constant: A quantity which doesn't change its value throughout a set of mathematical work is called a
constant. For example: a, b, c, d, e, α, β and 1, 10, 10000, etc.
Variable: A quantity which takes any numerical value in the mathematical operation is called a variable.
For example: x, y, z, u, v, w, t, etc.

1.2 Function
Let us consider, y = 2x + 5. When x = 0, y = 5, when x = 1, y = 7 and so on.
Here, we can see that for each value of x, there corresponds a unique value of y. Thus, it is a rule by
means of which we can calculate the values of y when the values of x are given or specified. In this case,
2 BUSINESS MATHEMATICS I

the value of y depends on the value of another variable x. Since the value of y is determined by the value
of x, we say that y is a function of x. It can be denoted by y = f (x) and read as y is a function of x.
Definition: If two variables x and y are so related that for every value of x in a given set, there
corresponds a unique value of y then y is called a function of x.
In other words, a function f from a set A to set B is a rule that assigns each element of A with exactly one
element of B. It is denoted by f : A → B.
If y = f (x) is a function, then x is independent variable and y is dependent variable. Also, y is the
image of x and x is the pre-image of y.
A function also can be analyzed as input-output device.
Input Function Output

Thus, a function is a rule that assigns each input value to one and only one output value.
The letters f, g, h, F, etc. are used to denote functions. Functions can be represented by a graph or by an
equation or in words or in a table. Let us consider an example as follows.
Example 1. Let A = {1, 2, 3}. A function f is defined on A by y = f (x) = x + 1. Express f by both table
and graph.
Solution
Given A = {1, 2, 3} and the function f is defined on A by y = f (x) = x + 1.
When x = 1, f (1) = 1 + 1 = 2 Graph
When x = 2, f (2) = 2 + 1 = 3 Y
When x = 3, f (3) = 3 + 1 = 4
Table 5
x (Input) 1 2 3
4 (3, 4)
y (Output) 2 3 4
3 (2, 3)

2 (1, 2)

X
O
1 2 3 4 5

1.3 Domain, Co-domain and Range of a Function


Let f : A → B be a function. The set A is known as the domain of f and the set B is known as the co-
domain of f. The set of all images of elements of A is known as the range of f and is denoted by f (A).
Thus, f (A) = {f (x) : x ∈ A} = Range of f. Clearly, f (A) ⊆ B = Co-domain of f.
Example 2. Let A = {1, 2, 3} and B = {1, 4, 9, 16}. Define f (x) = x2. Is f a function? Also, find domain,
co-domain and range.
f
Solution A B
Here, f (1) = 1, f (2) = 4, f (3) = 9.
Clearly, each element in A has a unique image in B, f is a 1 1

function. 2 4
3 9
Domain ( f ) = {1, 2, 3}, range ( f ) = {1, 4, 9} and
16
co-domain = {1, 4, 9, 16}.
Linear Functions, Equations, Inequalities and Its Applications UNIT 1 3

x
Example 3. Let A = {– 1, 0, 2, 4, 6} and a function f : A → R be defined by f (x) = x + 2. Find the range
of f.
Solution
x
Here, f (x) = x + 2 and A = {–1, 0, 2, 4, 6}.

–1 0
When x = –1, f (–1) = –1 + 2 = –1 When x = 0, f (0) = 0 + 2 = 0

2 1 4 2
When x = 2, f (2) = 2 + 2 = 2 When x = 4, f (4) = 4 + 2 = 3

6 3
When x = 6, f (6) = 6 + 2 = 4

⎧ 1 2 3⎫
∴ Range of f = ⎨–1‚ 0‚ 2‚ 3‚ 4⎬ .
⎩ ⎭

1.4 Various Types of Functions


1. One-One or One to One Function (Injective)
Let f : A → B be a function. The function f is said to be one-one function f
or an injective if different elements of A have different images in B. A B
Thus, f : A → B is one-one if and only if x1 ≠ x2 ⇒ f (x1) ≠ f (x2) for all
a 1
x1, x2 ∈ A. Equivalently f (x1) = f (x2) ⇒ x1 = x2 for all x1, x2 ∈ A.
b 2
Example: Let A = {a, b, c} and B = {1, 2, 3}. Let us define
c 3
f : A → B by f (a) = 1, f (b) = 2 and f (c) = 3. Clearly, f is a one to one
function from A to B, since different elements in A have different images in B.
2. Onto Function (Surjective)
Let f : A → B be a function. If every element in B has at least one pre- f
image in A then f is said to be an onto function (surjective). Thus, f : A → B A B

is a surjective if and only if for each y ∈ B, there exists x ∈ A such that


-1
f (x) = y. 1
1
Thus, f is onto if and only if range ( f ) = B = Co-domain ( f ). 2
4
Example: Let A = {–1, 1, 2, – 2}, B = {1, 4} and f : A → B be a function -2

defined by f (x) = x2. Clearly, f is onto since f (A) = {f (–1), f (1), f (2),
f (–2)} = {1, 4} = B.
3. Into Function f
A B
Let f : A → B be a function. If there exists at least one element in B
having no pre-image in A then f is said to be an into function. 3 1
Example : Let A = {3, 5, 7} and B = {1, 3, 5, 7}. Let f : A → B be 5 3
defined by f (x) = x – 2. Then, f (3) = 1, f (5) = 3 and f (7) = 5. Clearly, f is 7 5

a function from A to B. Here, 7 ∈ B, has no pre-image in A. So f is an 7

into function.
4 BUSINESS MATHEMATICS I

4. Many to One Function


f
Let f : A → B be a function. If two or more than two elements of A have
A B
the same image in B, then f is said to be many to one function.
Thus, f : A → B is a many-one function if there exist x1, x2 ∈ A such that -2 1
x1 ≠ x2, but f (x1) = f (x2). 1
Example: Let A = {–2, 1, 2, 3} and B = {1, 4, 9}. Define f : A → B by f 2 4
(x) = x2. Then, each element in A has a unique image in B. So f is a 3 9
function from A to B.
Here, f (–2) = 4, f (1) = 1, f (2) = 4 and f (3) = 9. Clearly, two elements namely 2 and –2 have the
same image 4 ∈ B. So f is many-one function. f
5. Bijective Function A B
A function which is both one-one and onto function is said to be
bijective. A bijective function is also known as a one-to-one x 1
correspondence. y 2
Example: Let A = {x, y, z} and B = {1, 2, 3}. Let us define f : A → B by z 3
f (x) = 1, f (y) = 2 and f (z) = 3. Clearly, f is a bijective function from A to
B, since different elements in A have different images in B and f (A) = B.
NOTE Let f : A → B. Then,
i. f is one-one ⇒ n(A) ≤ n(B).
ii. f is onto ⇒ n(B) ≤ n(A).
iii. f is both one-one and onto ⇒ n(A) = n(B).

1.5 Inverse Function


–1
Let f : A → B be both one-one and onto function. Then, f : B → A is a f
function which associates to each y of B, a unique x of A such that f (x) = y A B

is called the inverse function of f.


–1
∴ f (y) = x if and only if f (x) = y. x y

From above definition, we can say that if function f maps x into y then
function f –1 maps y back into x.
We note that domain of f –1 = range of f, range of f –1 = domain of f. f –1

Example 4. If f (1) = 3, f (2) = 5 and f (3) = 7, find f –1 (3), f –1(5) and f –1(7).
Solution f f –1
–1
f (1) = 3 ⇒ f (3) = 1 A B A B

f (2) = 5 ⇒ f –1(5) = 2
1 3 1 3
f (3) = 7 ⇒ f –1(7) = 3
2 5 2 5
3 7 3 7

We can see that inverse function reverses the inputs and outputs.
–1
Example 5. Let f : § → § defined by f (x) = 3x + 5. Find f .
Solution
Let y = f (x) = 3x + 5.
Interchanging x and y,
y = 3x + 5
Linear Functions, Equations, Inequalities and Its Applications UNIT 1 5

or, 3y = x – 5
x–5
or, y= 3 .
–1 x–5
∴ f (x) = 3 .

1.6 Composite Function


Let A, B, and C be three sets. Let f : A → B and g : B → C be two functions. Since f : A → B, for each x ∈ A
there exists a unique element f (x) ∈ B. Again, since g : B → C , corresponding to f (x) ∈ B, there exists a
unique element g(f (x)) of C. Thus, for each x ∈ A, there is a unique element g(f (x)) of C. Thus, from f and g,
we can define a new function from A to C. This function is called the composite of f and g denoted by gof : A
→ C and is defined by (gof ) (x) = g(f (x)), for all x ∈ A.
f g
A B C

x f (x) g (f (x))

gof

NOTE 1. If fog exists then gof may or may not exist and vice-versa.
2. In general, fog ≠ gof.
3. If gof is one-one then f is one-one.
4. If gof is onto then g is onto.
5. If f and g are one-one and onto then gof and fog are one-one and onto.

Example 6. Find gof and fog, if f : § → § and g : § → § are given by f (x) = 2x and g(x) = 3x2. Show
that gof ≠ fog.
Solution
We have Again
gof (x) = g (f(x)) fog (x) = f (g(x))
= g(2x) = f (3x2)
= 3(2x)
2
= 2(3x2)
= 3 × 4x2 = 6x2
= 12x2. Hence, gof ≠ fog.

Example 7. If f : § → § and g : § → § be defined by f (x) = x + 2, g (x) = 2 – x, find fog (5).


Solution
fog (x) = f (g(x)
= f (2 – x)
=2–x+2=4–x
∴ fog (5) = 4 – 5 = – 1.
6 BUSINESS MATHEMATICS I

Even and Odd Functions


If a function f satisfies f (–x) = f (x) for all x in its domain, then the function f is called an even function.
For example, f (x) = x2 is even function because
f (– x) = (– x)2 = x2 = f (x).
Similarly, a function f is called odd function if f (– x) = – f (x) for all x. For example, f (x) = x is odd
function as,
f (– x) = –x = – f (x).
A function could be neither odd nor even. For example f (x) = x2 + x is neither odd nor even.

 Illustrative Examples
Example 1. Let f : A → R be given by f (x) = 2 |x| + 3 where A = {– 2, 0, 1, 2}. Find the range of f.
Solution
When x = –2, f (– 2) = 2|– 2| + 3 = 2 × 2 + 3 = 7
When x = 0, f (0) = 2|0| + 3 = 3
When x = 1, f (1) = 2|1| + 3 = 2 × 1 + 3 = 5
When x = 2, f (2) = 2|2| + 3 = 2 × 2 + 3 = 7
∴ Range of f = {f (– 2), f (0), f (1), f (2)}
= {7, 3, 5, 7} = {3, 5, 7}.
x–1
Example 2. Let a function f : A → B be defined by f (x) = x + 2 with A = {–1, 0, 1, 2, 3, 4} and
⎧ 1 1 1 2⎫
B = ⎨–2, 1, –2, 0, 2, 4, 5⎬. Find the range of f.
⎩ ⎭
Solution
x–1
Here, the function f : A → B is defined by f (x) = x + 2 and A = {–1, 0, 1, 2, 3, 4} and
⎧ 1 1 1 2⎫
B = ⎨–2‚ –1‚ –2‚0‚ 2‚ 4‚ 5⎬.
⎩ ⎭
A B
–1 – 1
When x = –1, f (–1) = –1 + 2 = –2 –1 –2
0 1
0–1 1
When x = 0, f (0) = 0 + 2 = –2 1 –1/2
2 0
1–1 3 1/2
When x = 1, f (1) =1+2 =0
4 1/4
2–1 1 2/5
When x = 2, f (2) =2+2=4
3–1 2
When x = 3, f (3) =3+2 =5
4–1 1
When x = 4, f (4) =4+2 =2
⎧ 1 1 2 1⎫
∴ Range of f = ⎨–2‚ –2‚ 0‚ 4‚ 5‚ 2⎬ .
⎩ ⎭
Linear Functions, Equations, Inequalities and Its Applications UNIT 1 7

Example 3. Let f: § → § be defined by y = f (x) = 3x – 2. Find f -1(x) and hence, find f -1(1) and f -1(4).
Solution
Given y = 3x – 2
Interchanging x and y, we get
x = 3y – 2
or, x + 2 = 3y
x +2
or, y = 3
x +2
∴ f -1(x) = 3
1+2 3
Now f -1(1) = 3 = 3 = 1
4+2 4+2 6
f -1 (4) = 3 = 3 = 3 = 2.

Example 4. If functions f : R → R defined by f (x) = 2x + 1 and g : R → R defined by g(x) = x2 – 2. Find


the formulae for composite functions fog and gof and also verify that fog ≠ gof.
Solution
Given f (x) = 2x + 1 and g(x) = x2 – 2.
Now And,
fog(x) = f (g(x)) gof (x) = g(f (x))
= f (x2 – 2) = g(2x + 1)
= 2(x2 – 2) + 1 = (2x + 1)2 – 2
= 2x2 – 4 + 1 = 4x2 + 4x + 1 – 2
= 2x2 – 3 = 4x2 + 4x – 1
∴ fog(x) ≠ gof (x).

 Exercise 1.1
1. The following arrow diagram shows a function f defined from A to B. f
A B
a. Write the images of 1, 2, 3 and 4.
b. Write the pre-images of a, b, c, d, e. 1 a
c. Write domain, range and co-domain of the function f. 2 b
3 c

2. If f(x) = x2 – 2x + 2, find the value of 4 d


e
a. f (0) b. f (– 1)
c. f (– 2 ).
3. Find the range of the following functions f : A → B.
x–1 x
a. A = {–1, 0, 1, 2} and f (x) = x + 2 . b. A = {–1, 0, 1, 2} and f (x) = x + 3 .

4. If f : x → 3x + b and f (2) = 12, find the value of b.


5. Determine the inverse of following functions.
a. y = 2x – 3 b. y = 3x + 5
1
c. y = 2 x + 3 d. y = x3 + 1
8 BUSINESS MATHEMATICS I

6. If f (x) = 2x + k and f –1(4) + f (2) = 7, find the value of k.


4x – 2
7. If f (x) = 3x – 7, g(x) = 3 and f –1(x) = g(x), find the value of x.

8. Find the composite functions fog and gof of the following functions, where f : § → § and g : § → §.
a. f (x) = 2x + 1 and g = x + 2 b. f (x) = 4x + 7 and g(x) = 5x – 2
2
c. f (x) = 2x + 3 and g(x) = x .
x+1 x–5
9. If f (x) = 2 , g(x) = 2 and fog –1 (x) = 6, find the value of x.
3x + 1
10. If f (x) = 2x + 1 and g(x) = 2 are two functions and if fof (x) = g –1(x), find the value of x.

11. Let f : § → § be defined by f(x) = ax + b, a (≠ 0) and b are real numbers. Find f –1 and show that
f (f –1(x)) = f –1(f (x)) = x.

ANSWERS
1. (a) a, c, c, e. (b) 1, No, 2 and 3, No, 4
(c) Domain = {1, 2, 3, 4}, Range = {a, c, e}, Co-domain = {a, b, c, d, e}
2. (a) 2 (b) 5 (c) 2(2 + 2)

3. (a) Range = { 1
}
1
–2‚ –2‚ 0‚ 4
1
{ 1 2
(b) Range = –2‚ 0‚ 4‚ 5}
4. b=6
x+3 x–5 3
5. (a) 2 (b) 3 (c) 2(x – 3) (d) x–1

6. k=2 7. 3
8. (a) fog = 2x + 5 and gof = 2x + 3 (b) fog = 20x – 1 and gof = 20x + 33
(c) fog = 2x2 + 3 and gof = (2x + 3)2
x–b
9. 0 10. x = –1 11. a

1.7 Linear Functions and their Graphs


Linear Functions
A function of the type y = ax + b, where x is any real variable and a and b are constants and a ≠ 0 is called
linear function. A linear functions is a first degree function and always gives a straight line. Some
examples of linear functions are (i) y = 3x – 2 (ii) y = 4x + 3, etc.
Graphical Representation Y

7
Consider a linear function, y = f (x) = 2x + 3.
6 y = 2x + 3
x 0 1
5
y 3 5
4
The graph of this function is shown along side.
3

X
O
1 2 3 4 5
Linear Functions, Equations, Inequalities and Its Applications UNIT 1 9

Example 1. Express graphically the following linear functions.


a. y = f (x) = x b. x=3 c. y = 4.
Solution
a. y = f (x) = x Y

x 0 1 3
y 0 1
2
y=x
The graph of this function is shown along side.
1

X
O 1 2 3
b. x=3 Y
We can write x + 0 × y = 3
3
x 3 3
2
y 0 1 x=3
1
The graph shows that the function is a straight
line 3 units right of y-axis and is parallel to it. X
O 1 2 3

c. y=4 Y
We can write 0 ⋅ x + y = 4 4
y=4
x 0 1
3
y 4 4
2
The graph shows that, the function is a straight
line parallel to x-axis and 4 units above it. 1

X
O 1 2 3

1.8 Linear Equations


All first degree equations are called linear equations. Some linear equations are given below:
i. ax = b
ii. ax + by + c = 0
iii. x – 3y = 0, etc.
From above examples, we found the linear equation consists of variables x, y, the coefficients and the
constant of real numbers.
Linear equations may be of having one variable, two variables, three variables and more.
Linear Equation with One Variable
b
ax + b = 0 so that: x = –a , be the unique solution. For example if 3x + 6 = 0, then 3x = –6.

∴ x = – 2 be its unique solution.


Linear Equation with Two Variables
Consider a linear equation with two variables x and y :
ax + by + c = 0
10 BUSINESS MATHEMATICS I

In above equation if b ≠ 0, then


by = – ax – c
a c
∴ y=–bx–b.

a c
The graph of the function is a line with slope m = – b and y-intercept = C = – b . [The slope and
y-intercept are discussed in 1.9]
For example: If 3x + 2y = 6 then 2y = –3x + 6.
3 3
or, y = – 2 x + 3; having its slope = m = – 2 and y-intercept = C = 3.

We can plot a graph by putting different values of x and hence, the corresponding values of y:
Y
x 0 2
y 3 0 5

3
3x + 2y = 6
2

X
O
1 2 3 4 5

1.9 Slope and Intercept of a Line


Slope of a Line
The slope of a straight line is the steepness of a line with respect to horizontal axis. Let two points in the
plane be P(x1, y1) and Q(x2, y2). Then, the increments Δx = x2 – x1 and Δy = y2 – y1 are the run and the rise
respectively between P and Q. Two such points
Y
always give a unique straight line passing through
both of them. We denote this straight line by PQ.
(Δx is read as 'delta x' and means change in x and Q(x2, y2)
y2
Δy is read as 'delta y' and means change in y). y2 – y1
For any non-vertical line PQ, the ratio P(x1, y1)
y1
rise y2 – y1 Δy
m = run = x – x = has the same value for x2 – x1
2 1 Δx
every choice of two points P and Q on that line.
This constant value is called slope of line PQ. The X
O
figures shows the rise and run between two points x1 x2
P and Q.
The slope of vertical line is undefined.
Thus, the ratio of the change in y-coordinates to the corresponding change in x-coordinates measures the
slope of any non-verbal line. The slope of a line can also be described as the number of units by which y
changes when x increases by 1 unit.
NOTE A straight line has one and only one slope.
Linear Functions, Equations, Inequalities and Its Applications UNIT 1 11

If θ be the angle made by a straight line with positive x-axis then m = tan θ is the slope of the straight line.
The slope describes the direction and steepness of a line. We discuss them as follows:
Y
1. The slope is positive if the line is rising from left to right.
Δy
m= > 0.
Δx

X' X
O

Y'
Y
2. The slope is negative if the line is falling from left to right.
Δy
m= < 0.
Δx

X' X
O

Y'
Y
3. The slope of a horizontal line is zero because Δy = 0.
Δy
m= = 0.
Δx

X' X
O

Y'
Y
4. The slope of a vertical line is undefined because Δx = 0.
Δy
m= = ∞ (infinity).
Δx

X' X
O

Y'

Example 2. Find the slope of a line


a. making an angle of 60° with positive x-axis.
b. joining the two points (3, 5) and (–1, 2).
Solution
Here,
a. θ = 600
∴ The slope of a line (m) = tan θ = tan 600 = 3.
12 BUSINESS MATHEMATICS I

b. (x1, y1) = (3, 5) and (x2, y2) = (–1, 2)


y2 – y1 2–5 –3 3
∴ The slope of a line (m) = x – x = –1 –3 = –4 = 4 .
2 1

Intercepts of a Line Y

Let a line AB cut x-axis and y-axis at the points A and B


respectively. The x-coordinate of point A is called x-intercept B
and y-coordinate of point B is called y-intercept of line AB.
In this figure, OA is x-intercept and OB is y-intercept.

y-intercept
NOTE 1. To find y-intercept of a line, put x = 0 in the equation of line.
2. To find x-intercept of a line, put y = 0 in the equation of line.

A
X
O
x-intercept

Example 3. Find the intercept made by the line 3x + 4y = 12 on the coordinate axes and graph it.
Solution
Given
3x + 4y = 12 . . . (i) Y
Put x = 0 in equation (i),
3 × 0 + 4y = 12 5

∴ y=3 4
Hence, y-intercept is 3. 3
Again, put y = 0 in equation (i), 3x + 4y = 12
2
3x + 4 × 0 = 12
∴ x=4 1

Hence, x-intercept is 4. X
O
The graph of the line is given alongside. 1 2 3 4 5
Different forms of equation of straight line:
1. Slope intercept form
y = mx + c [Here, slope = m, y-intercept = c]
2. Double intercept form
x y
a+b=1 [Here, x-intercept = a, y-intercept = b]
3. Point slope form
y – y1 = m(x – x1) [Here, slope = m, point (x1, y1)]
4. Two points form
y2 – y1
y – y1 = x – x (x – x1) [Here, points are (x1, y1) and (x2, y2)]
2 1

Example 4. Find the slope of the line passing through (2, 3) and (5, 9).
Solution
Here, (x1, y1) = (2, 3) and (x2, y2) = (5, 9).
y2 – y1 9 – 3 6
∴ Slope (m) = x – x = 5 – 2 = 3 = 2.
2 1
Linear Functions, Equations, Inequalities and Its Applications UNIT 1 13

Example 5. Find the slope and y-intercept of the function 3y – 6x + 15 = 0.


Solution
Given function is
3y – 6x + 15 = 0
or, 3y = 6x – 15
6x – 15
or, y = 3
∴ y = 2x – 5 . . . (i)
Comparing equation (i) with y = mx + c, we get slope = m = 2 and y-intercept = c = –5.

 Exercise 1.2
1. Find the slope of the straight line passing through the following points.
a. (1, 1) and (2, 2) b. (3, 2) and (3, 3).
2. Find the slope and y-intercept of the following functions.
a. ax + by + c = 0 b. y = mx c. y = c
d. 3x – 2y = 3 e. 4x + y = 8
3. Find the intercepts made on the axes by the following straight lines. Find their slopes also.
a. x – 2y – 2 = 0 b. 3x – 2y – 6 = 0
4. Find equation of straight line in the following cases.
1
a. Slope = m = 3 and y-intercept = c = –2 b. Slope = m = 3 and y–intercept = c = – 4.
5. a. Find the equation of a straight line making 9 and 15 as the x–intercept and y–intercept
respectively.
b. Find the equation of straight line with x–intercept 2 and y–intercept 3.
6. Find equation of straight line in the following cases.
1
a. Slope = m = 2 and a point (3, 2) b. Slope = m = 3 and a point (3, 1).
7. Find equation of straight line in the following cases.
a. Passing through the points (2, 3) and (4, 1) b. Passing through the points (–1, 1) and (2, 3).
8. Solve:
2x x–1 2x + 7 x – 4 1
a. x + 2 = 2(x + 3) b. 5 +1= 2 c. 3 = 6 +2
9. Graph the following:
a. x + y = 4 b. y = 3x + 4 c. y = 3 – 2x

ANSWER
1. (a) 1 (b) ∞
a c 3 3
2. (a) – b , – b (b) m, 0 (c) 0, c (d) 2 , –2

(e) –4, 8
1 3
3. (a) a = 2, b = –1, m = 2 (b) a = 2, b = –3, m = 2

4. (a) x – 3y = 6 (b) 3x – y – 4 = 0
5. (a) 5x + 3y = 45 (b) 3x + 2y = 6
6. (a) 2x – y = 4 (b) x – 3y = 0
7. (a) x+y=5 (b) 2x – 3y + 5 = 0
8. (a) –4 (b) 15 (c) –5
14 BUSINESS MATHEMATICS I

9. (a) Y (b) Y B
B
4 (0, 4) 4 (0, 4)

3 3

2 2

1 1
(4, 0) (–4/3, 0)
X' X X' X
O O
-2 -1 1 2 3 4 5 -3 -2 -1 1 2 3 4
-1 -1
A
-2 A
-2

Y' Y'

(c) Y
B
4
(0, 3)
3

1
(3/2, 0)
X' X
O
-3 -2 -1 1 2 3 4
-1
A
-2

Y'

1.10 Linear Inequalities and their Properties


Inequalities
Consider a real number a. This number 'a' may be positive, negative or zero. If a is positive we write
a > 0, if a is negative, we write a < 0 and if a is equal to zero, we write a = 0.
Similarly, for any two numbers a, b ∈ R,
i. a is said to be greater than b i.e. a > b, if a – b > 0.
ii. a is said to be lesser than b i.e. a < b, if a – b < 0.
iii. a is said to be equal to b i.e. a = b, if a – b = 0.
The relations in (i) and (ii) are known as inequalities and (iii) is known as equality.
A mathematical statement containing the sign > or < or ≥ or ≤ is called an inequality.
Properties of Inequalities
The following are the properties of inequalities
Let a, b and c be three real numbers.
a. If a > b then a + c > b + c b. If a > b, then a – c > b – c
c. If a > b, then ca > cb if c > 0 d. If a > b, then ca < cb if c < 0
e. If a < b, then a + c < b + c f. If a < b, then a – c < b – c
g. If a < b, then ca < cb if c > 0 h. If a < b, then ca > cb if c < 0
i. If a > b, then – a < – b j. If a > 0, then – a < 0.
Linear Functions, Equations, Inequalities and Its Applications UNIT 1 15

Example 1. Solve: 6 + 4x ≤ 12.


Solution
6 + 4x ≤ 12
Adding – 6 on both sides
– 6 + 6 + 4x ≤ 12 – 6
or, 4x ≤ 6
Dividing both sides by 4
4x 6
or, 4 ≤ 4
3
∴ x ≤ 2.

Intervals
Let 'a' and 'b' be any two points on the real line with a < b. The set of all points between a and b is called
an interval. The points 'a' and 'b' are called the end points of the interval. An interval may or may not
contain end points. There are four types of intervals. They are:

Closed interval : [a, b] = {x : a ≤ x ≤ b}


a b

Open interval : (a, b) = {x : a < x < b}


a b

Right open interval : [a, b) = {x : a ≤ x < b}


a b

Left open interval : (a, b] = {x : a < x ≤ b}


a b

Some examples of intervals are as follows:

The figure alongside represents the half closed interval [0, ∞). This is
–2 –1 0 1 2 3
written as [0, ∞) = {x: 0 ≤ x < ∞}.

The figure alongside represents the closed interval – 2 ≤ x ≤ 2,


–2 –1 0 1 2 3
i.e., [–2, 2] = {x : – 2 ≤ x ≤ 2}.
The figure alongside represents the half open interval x < – 1,
–2 –1 0 1 2
i.e., (–∞, – 1) = {x : – ∞ < x < – 1}.
The real number line is represented by
§= (–∞, ∞) = {x : – ∞ < x < ∞}.
NOTE '( )' is used for the open interval, where end points are excluded and '[ ]' is used for the closed interval
where the end points are included.

1.11 Absolute Value of a Real Number


The absolute value or modulus of a real number x, denoted by | x |, is a non-negative number defined as:

⎧ x if x > 0
| x | = ⎨ –x if x < 0 .
⎩ 0 if x = 0
16 BUSINESS MATHEMATICS I

Graph of Absolute Value


The graph of absolute value y = | x | can be plotted by the following way:
Y
⎧ x if x > 0
Take y = | x | = ⎨ –x if x < 0 5
⎩ 0 if x = 0 4
x –2 –1 0 1 2 3
y 2 1 0 1 2 (-2, 2)
2
(2, 2)
1
Properties of Absolute Values (-1, 1) (1, 1)
X' X
O
1. For any real number x; -5 -4 -3 -2 -1 (0, 0) 1 2 3 4 5 6

a. –x ≤ | x | Y'
b. x ≤ | x | Graph of y = | x |
2. If x and y be any two real numbers then:
a. | x + y | ≤ | x | + | y | (Triangle inequality)
b. | x – y | ≥ | x | – | y |
c. | xy | = | x | | y |
x |x|
d. ⎪y⎪ = | y | , y ≠ 0
⎪⎪
3. For any positive real number a and a real number x,
a. | x | < a ⇒ – a < x < a
b. –a < x < a ⇒ | x | < a.
Example 2. Find the value of
a. |–2| b. 8–|–3|
Solution
a. | – 2 | = – (–2) = 2 b. 8 – | – 3 | = 8 – 3 = 5.
Example 3. If x = – 4 and y = 3 then verify that
a. ⎢x – y ⎢ ≥ ⎢x ⎢ – ⎢y ⎢ b. x+y ≤ x + y .

x |x|
c. ⎢xy ⎢ = ⎢x ⎢⋅ ⎢y ⎢ d. y = |y|

Solution
a. Now b. |x + y| = |– 4 + 3|
|x| – |y| = |– 4| – |3| = |−1|
= – (– 4) – 3 =1
=4–3 |x| + |y| = |– 4| + |3|
=1 =4+3
|x – y| = |– 4 – 3| =7
= |– 7| ∴ |x + y| < |x| + |y|.
= – (– 7)
=7
∴ |x – y| > |x| – |y|.
Linear Functions, Equations, Inequalities and Its Applications UNIT 1 17

c. |xy| = |(–4) × 3| ⎪x⎪ = ⎪– 4⎪ = 4


d.
= |–12| ⎪y⎪ ⎪ 3⎪ 3
= 12 |x| |– 4| 4
|y| = |3| = 3
|x| ⋅ |y| = |–4| |3|
=4×3 ∴ ⎪x⎪ |x|
= |y|.
= 12 ⎪y⎪
∴ | xy | = |x| ⋅ |y|.
Example 4. Rewrite the following without absolute sign
a. |x – 1| < 6 b. |3x – 2| ≤ 7.
Solution
a. |x – 1| < 6 b. |3x – 2| < 7
⇒ −6<x–1<6 [By definition] ⇒ − 7 < 3x – 2 < 7
Adding 1 to each side. Adding 2 to each side.
⇒ −6+1<x–1+1<6+1 ⇒ − 7 + 2 < 3x + 2 – 2 < 7 + 2
⇒ − 5 < x < 7. ⇒ −5≤3x≤9
Dividing each side by 3.
5
⇒ − 3 ≤ x ≤ 3.

Example 5. Rewrite the following using absolute value sign


a. –3 < x < 5 b. –1<x<7 c. – 3 ≤ x ≤ 8.
Solution
a. –3<x<5 b. –1<x<7
Adding – 1 to each side Adding –3 to each side, we get
⇒ –3–1<x–1<5–1 ⇒ –1–3<x–3<7–3
⇒ –4<x–1<4 ⇒ –4<x–3<4
∴ |x – 1| < 4. ∴ |x – 3| < 4.
[∵ – a < x < a ⇒ |x| < a] [∵ – a < x < a ⇒ |x| < a]
c. –3<x<8 Alternative Method:
5 –3<x<8
Adding –2 to each side, we get
Multiplying each side by 2
5 5 5 −6 ≤ 2x ≤ 16
⇒ –3–2<x– 2 <8–2
Again, adding −5 to each side
– 11 5 11 −11 ≤ 2x − 5≤ 11
⇒ 2 <x –2< 2
∴ |2x − 5| ≤ 11.
– 11 2x – 5 11
⇒ 2 < 2 < 2
⇒ – 11 ≤ 2x – 5 ≤ 11
∴ |2x – 5| < 11.

NOTE To write a ≤ x ≤ b using absolute value sign we add − (a +2 b) to each side.


18 BUSINESS MATHEMATICS I

 Exercise 1.3
1. Find the values of

a. |–7| + 2 b. |–7 + 2| c. 4 – |–2| d. ⎪2 + 3⎪.


⎪ –5 ⎪
2. a. If 3x – y < 3y – x, prove that x < y.
b. If 0 ≤ 3x + 9 ≤ 27, prove that – 3 ≤ x ≤ 6.
c. If – 5 < 7x + 9 < 30, prove that – 2 < x < 3.
d. If – 8 ≤ 2x + 2 < – 2, prove that – 5 < x < –2.
e. If – 7 ≤ 2x + 5 ≤ 7, prove that – 6 ≤ x ≤ 1.
f. If – 10 < 5x + 10 < 5, prove that – 4 < x < – 1.
3. If x = – 3, y = 5, verify that:
a. |x+y|≤|x|+|y| b. |x|–|y|≤|x–y|
x |x|
c. | xy | = | x | | y | d. y = |y|.
4. Rewrite the following without using absolute value sign.
a. |x – 2| < 6 b. |x + 5| ≤ 7 c. |x + 2| ≤ 5
d. |2x + 5|≤ 7 e. |3 − 5x| ≤ 2.
5. Rewrite the following using modules sign.
a. –3 < x < 3 b. –5 ≤ x ≤ 1 c. –4 ≤ x ≤ 2
d. – 4 ≤ x ≤ 1 e. –3 ≤ x ≤ – 2 f. – 5 < x < – 2.
6. Graph the following:
a. y = 2| x | b. y = –3 | x |.

ANSWERS
1. (a) 9 (b) 5 (c) 2 (d) 1
4. (a) –4 < x < 8 (b) –12 ≤ x ≤ 2 (c) –7 ≤ x ≤ 3 (d) – 6 < x < 1
1
(e) 5 ≤ x ≤ 1

5. (a) |x| < 3 (b) | x + 2| ≤ 3 (c) |x + 1| ≤ 3 (d) |2x + 3| < 5


(e) |2x + 5| < 1 (f) |2x + 7| < 3
6. (a) (b) Y
Y
y= 2|x|
1
4 O(0, 0)
X' X
3 -3 -2 -1 1 2 3 4
-2
2
(-1, -3) -3 (1, -3)
1 -4
(0, 0)
X' X -5
O
-3 -2 -1 1 2 3 (-2, -6) -6 (2, -6)
-1
-7
-2
y = – 3|x|
Y' Y'
Linear Functions, Equations, Inequalities and Its Applications UNIT 1 19

Application of Linear Functions


Linear function is that function whose graph is a straight line. It is popular in management and economics
because it is simple and easy to handle. Many economic laws can be represented by linear function. We
now define and discuss various terminologies used in management and economics.

1.12 Demand, Supply, Cost, Revenue and Profit Function


Demand
Demand is a consumer's desire to purchase goods and services, and willingness to pay a price for goods
and services. If all other factors remain same, an increase in the price of goods or services will decrease
the quantity demanded, and vice-versa.
Demand Function
There are several variables that influence the demand for a good. These variables may be quantity
demanded, price, income of consumer, price of complementary goods, price of substitute goods, fashion
or taste of the consumer, level of advertising, etc. Here, we assume that the other factors (variables)
remain constant and suppose that the demand function is the function of price. The simplest model is
Q = f (P).
This means the quantity demanded depends on price only if other variables remain constant.
For example, the demand for a good X is given by Q = 600 – 4P. This equation describes the law of
demand. According to basic economics hypothesis, there is a negative relationship between quantity
demanded and price. In the adjoining figure, the price P is on the horizontal axis and quantity Q is on the
vertical axis.
Y

600

500
Q = 600 – 4P
400
Quantity (Q)

300

200

100

X
O
30 60 90 120 150
Price (P)
On the other hand, if Q is plotted on the horizontal axis and P is on the vertical, we calcaulate inverse
demand function
P = f –1 (Q).
20 BUSINESS MATHEMATICS I

The inverse demand function can be calculated as follows:


Q = 600 – 4P.
or, 4P = 600 – Q
∴ P = 150 – 0.25Q
Y

150

120
Price (P)

P = 150 – 0.25Q
90

60

30

X
O
100 200 300 400 500 600
Quantity (Q)
For our convenience, we do not distinguish between the terms demand function and inverse demand
function.
Equation of Demand Function
The demand function P = f (Q) can be modeled by the linear equation
P = a – bQ . . . (i)
where a and b are (positive) constants. This is the equation of straight line. The general equation of
straight line is y = mx + c.
Thus, m = –b, c = a.
a > 0: The vertical intercept of the demand function is positive. The value of a depends on the size of the
terms on the R.H.S. of the general demand function, except Y
the price of the good itself.
(–b) < 0: The slope of the demand function is negative. The a (Vertical intercept)
negative slope indicates that price drops by b units for each
unit increase in quantity. P = a – bQ
Price (P)

a Slope = –b
When P = 0, from (i) Q = b .
a
Thus, the horizontal intercept occurs at (P, Q) = ⎛0‚ b⎞
⎝ ⎠
When Q = 0, from (i), P = a. X
O a
Thus, the vertical intercept occurs at (P, Q) = (a, 0). b
(Horizontal intercept)
Supply Quantity (Q)

Supply is the total amount of a certain goods or services which are available to consumers.
Supply Function
Supply function is a mathematical function or expression of the relationship between quantity demanded
of a product or service, its price and other associated factors such as input costs, price of other goods,
number of producers in the market, available technology, etc.
Linear Functions, Equations, Inequalities and Its Applications UNIT 1 21

The simplest model for the supply function can be written as Q = f (P). There is a positive relationship
between quantity supplied and price. This means that when the price of a good increases, the quantity
supplied also increases when all other variables remaining constant. This is the law of supply.
Equation of Supply Function
The supply function P = g(Q) can be modeled as
P = c + dQ . . . (i)
Y
where c and d are constants.
c > 0: The vertical intercept is positive.
d > 0: The slope of the supply functions is S

Price (P)
positive. Price increases by d units for every unit
increase in the quantity supplied. P = c + dQ
Slope = d
c
When P = 0, from (i) Q = – d . c (Vertical intercept)

Thus, the horizontal intercept occurs at X


c a O
(P, Q) = ⎛0‚ – d⎞ –b
⎝ ⎠ (Horizontal intercept) Quantity (Q)
When Q = 0, from (i), P = c.
Thus, the vertical intercept occurs at (P, Q) = (c, 0).
A minus quantity is not meaningful economically. Many economists illustrate the supply function in first
quadrant.
Example 1. The demand function is given by P = 200 – 0.5Q.
a. Find the slope. Explain its meaning.
b. What is the demand when P = 0?
c. What is the price when Q = 0?
d. Plot the demand function P = 200 – 0.5Q for 0 ≤ Q ≤ 400.
e. Find an expression for the demand function in the form Q = f (P) and plot it.
Solution
a. Given
P = 200 – 0.5Q . . . (1)
This is in the form of P = a + (–b)Q (i.e. y = c + mx).
Thus, slope = –0.5
This means that the price drops by 0.5 units for each successive unit increase in
quantity demanded.
b. When P = 0, from equation (1),
0 = 200 – 0.5Q
or, 0.5Q = 200
∴ Q = 400
That is the horizontal intercept is at Q = 400, P = 0.
c. What Q = 0, from equation (1) P = 200.
22 BUSINESS MATHEMATICS I

The vertical intercept is at P = 200.


d. From (b) and (c), we can plot the demand function P = 200 – 0.5Q for 0 ≤ Q ≤ 400 as
follows:
Vertical intercept (a) = 200 P

200

160 Slope = –b = –0.5

120 P = 200 – 0.5Q

80

40

Q
O
100 200 300 400 500

Horizontal intercept (ab) = 400


e. Given
P = 200 – 0.5Q
or, 0.5Q = 200 – P
or, Q = 400 – 2P . . . (2)
When P = 0, Q = 400
When Q = 0, P = 200
Q
= 400

200
()

400
a
b

1
Slope = –b = –2
Vertical intercept

300
Q = 400 – 2P
200

100

P
O
40 80 120 160 200
Horizontal intercept (a) = 200

Example 2. The supply function is given by P = 20 + 0.5Q.


a. Find the slope and intercepts and interpret them.
b. Plot the supply function for 0 ≤ Q ≤ 200.
Solution
a. Given
P = 20 + 0.5Q . . . (i)
The slope is 0.5. This means that the price increases by 0.5 units for every successive
unit increase in quantity supplied.
When Q = 0, from (i) P = 20.
Thus, the vertical intercept is c = 20.
Linear Functions, Equations, Inequalities and Its Applications UNIT 1 23

This means that the firm will supply no units at P ≤ 20.


Again, when P = 0, from (i)
0 = 20 + 0.5Q
∴ Q = –40
Thus, the horizontal intercept is at Q = –40, P = 0; however Q = –40 is not
economically meaningful.
b. To plot the supply function in the range 0 ≤ Q ≤ 200, we choose various values of Q
within this range and calculate the corresponding values for P.
Quantity (Q) Price (P)
0 20
40 40
80 60
120 80
160 100
200 120
Y

120

100

80
P = 20 + 0.5Q
Price

60
Slope = 0.5
40

20

X
O
40 80 120 160 200
Quantity

Cost
The monetary value of goods and services that producers and consumers purchase is called cost. To
produce goods and services for sale in the market, companies incur costs when they employ inputs such
as capital and labour. The total cost of producing goods consists of variable cost and fixed cost.
Variable cost (VC): Variable cost are the cost paid to the variable input. Thus, variable costs vary with
the level of output. The inputs include labour, materials, land and buildings, etc.
Fixed cost (FC): Fixed cost are the costs of the fixed assets those are not related to the level of output.
For example, rent of house, cost of machinery, etc.
Total cost function: Total cost (TC) is the sum of fixed costs and variable costs.
TC = FC + VC
Total cost refers to the total expenses incurred in reaching a particular level of input.
Cost function
Average cost function = Total Quantity
24 BUSINESS MATHEMATICS I

Example 3. The ABC company finds that the variable cost per unit of output is Rs. 10 and fixed cost is
Rs. 20.
a. Write the equation of the total cost function.
b. Graph the cost function.
Solution
a. Let Q be the number of outputs of the company.
FC = Rs. 20
VC = Q × Rs. 10 = Rs. 10Q.
Total cost function (TC) = VC + FC
∴ TC = 10Q + 20
b.
Quantity Total Cost Function Point
(Q) (TC = 10Q + 20) (Q, TC)
0 20 (0, 20)
1 10 × 1 + 20 = 30 (1, 30)
2 10 × 2 + 20 = 40 (2, 40)
3 10 × 3 + 20 = 50 (3, 50)
4 10 × 4 + 20 = 60 (4, 60)
5 10 × 5 + 20 = 70 (5, 70)
Y

70

60 TC = 10Q + 20

50

40
Cost (C)

30

20 FC = 20

10

X
O
1 2 3 4 5 6
Quantity (Q)
Revenue
Revenue is the income that a firm receives from the sale of a good or service to its customers. The price
of a good multiplied by the number of units sold gives the total revenue function.
Mathematically,
Total Revenue Function (TR) = P ⋅ Q
Total revenue function is linear if the price of each unit sold is same. That is each firm is a price-taker,
which is denoted by a horizontal demand function. A price-taking firm is a perfectly competitive firm.
Revenue function
Average revenue function = Total Quantity
Linear Functions, Equations, Inequalities and Its Applications UNIT 1 25

Example 4. A firm sold each article produced at Rs. 10. Find the total revenue function and graph it.
Solution
Total revenue function (TR) = Price of good × Number of units sold
= Rs. 10 × Q
= Rs. 10Q
where Q = No. of units sold.
This total revenue function is represented by a straight line with slope = 10 and
y–intercept = 0.
Quantity (Q) TR = 10Q Points (Q, TR)
0 10 × 0 = 0 (0, 0)
1 10 × 1 = 10 (1, 10)
2 10 × 2 = 20 (2, 20)
3 10 × 3 = 30 (3, 30)
4 10 × 4 = 40 (4, 40)
5 10 × 5 = 50 (5, 50)

60

50 TR = 10Q
Total Revenue (TR)

40

30

20

10

X
O
1 2 3 4 5 6
Quantity (Q)

Profit Function
The difference between total revenue function (TR) and total cost function (TC) is called profit function.
It is denoted by π. Thus,
π = TR – TC.
If π > 0, there will be profit and if π < 0, there will be loss.
If π = 0 at a certain point, then that point is called break even point. Thus, at break even point, we must
have TR = TC. If TR < TC then there is loss and if TR > TC then there is profit.
26 BUSINESS MATHEMATICS I

Break Even Point


The point at which total costs and total revenues are equal is called break even point. The break even
point can be described as a point where profit is zero.
Mathematically,
Profit (π) = 0
or, TR – TC = 0
∴ TR = TC Y

Revenue

Break Even Point


Cost and Revenue

Profit
(B.E.P.) Cost

Variable Cost

Loss

Fixed Cost

X
O
Quantity

Market equilibrium condition


The market is said to be in equilibrium condition if the quantity demanded is equal to the quantity
supplied i.e. Qd = Qs.
Example 5. In manufacturing a product, a firm bears a fixed annual cost of Rs. 100,000. In addition,
cost of each unit produced is Rs. 10. If C represents the total annual cost in rupees and x be
the number of units produced during a years. Determine the total annual cost function,
C = f (x).
Solution
We know that
Total cost = Fixed costs + Variable costs
C = f (x) = 100,000 + 10x.
Example 6. Nepal Brewery company finds that the variable cost per unit of output is Rs. 5 and fixed
cost is Rs. 20,000. If the output is sold for Rs. 10, find
a. the total cost function b. the revenue function
c. the profit function d. the break-even point.
Solution
Let C(x), R(x), and π(x) be the total cost function, revenue function and profit function
respectively for the output x. Then
a. Total cost function = C(x) = 5x + 20,000
b. Revenue function = R(x) = 10x
Linear Functions, Equations, Inequalities and Its Applications UNIT 1 27

c. Profit function = π(x) = R(x) – C(x) = 10x – (5x + 20,000)


= 5x – 20,000.
d. For break-even point π(x) = 0
or, 5x – 20,000 = 0
or, 5x = 20,000
20‚000
or, x = 5
∴ x = 4,000 units.
At the output level of x = 4,000 units, there will be break-even.
Example 7. Suppose a toy manufacturer has fixed cost of Rs. 50,000. In addition, there are variable
costs of Rs. 100 per toy.
a. Find the total cost function at a production level of x toys.
b. How many toys may be produced at a cost of 100,000 Rupees?
c. Suppose the selling price of each toy is Rs. 200, find the profit function.
d. Profit by selling 1,000 items.
Solution
Let C(x), R(x) and π(x) be the cost, revenue and profit function respectively. Then
a. Total cost function = C(x) = 50,000 + 100x
b. Total cost: C(x) = 100,000
Then, 50,000 + 100x = 100,000
or, 100x = 100,000 – 50,000
or, 100x = 50,000
∴ x = 500
c. Selling price of each toy is Rs. 200.
Revenue of x toys = R(x) = 200x.
Profit function = π(x) = R(x) – C(x)
= 200x – (50,000 + 100x)
= 100x – 50,000.
d. When x = 1,000,
π (1,000) = 100 × 1,000 – 5,000
= Rs. 50,000
Example 8. Find the equilibrium price and quantity of the following demand and supply function.
Qd = 20 – 2P and Qs = 4P – 4
Solution
Since the market is in equilibrium,
Quantity demanded = Quantity supplied
or, 20 – 2P = 4P – 4
or, 20 + 4 = 4P + 2P
or, 6P = 24
24
∴ P= 6 =4
Therefore, Price = P = 4 and
Demand function = supplied function = Q = 20 – 2 × 4 = 20 – 8 = 12.
28 BUSINESS MATHEMATICS I

15
Example 9. The demand function is: Q = 4 – P and the average cost function is AC = 2Q – Q . Obtain
the total cost function, the revenue function, the profit function, the break even quantity and
hence, the profit for 2 units of item sold, also the price.
Solution
Given
Demand function: Q = 4 – P
Then,
P =4–Q
We know
R =P⋅Q
= (4 – Q) ⋅ Q
or, R = 4Q – Q2
15
Average cost function: AC = 2Q – Q
We know
Total cost
Average cost = No. of items (Q)
15
or, ⎛2Q – Q ⎞ Q = total cost
⎝ ⎠
∴ Total cost function: C = 2Q2 – 15.
We know
Profit function (π) = R – C
= (4Q – Q2) – (2Q2 – 15)
∴ π = 4Q – 3Q2 + 15 . . . (i)
When Q = 2 units, then from (i)
π(2) = 4 × 2 – 3 × 22 + 15
= 8 – 12 +15
∴ π(2) = 11
And
P =4–Q
=4–2
∴ P = 2.

 Exercise 1.4
1. The demand function of a commodity is P = 50 – 2Q. Find the value of
a. P when Q = 5
b. Q when P = 10.
2. A firm has the demand function in the form P = a – bQ. The number of units demanded is 60 when
the price is Rs. 40 and 50 when the price is Rs. 60. Find the values of a, b, and the demand function.
1
3. A linear demand function has slope –2 . The price is $50 per unit when the quantity demand is 10
units. Find the demand function. Find the intercepts on both axes. What is the price if quantity
demand is 50 units?
Linear Functions, Equations, Inequalities and Its Applications UNIT 1 29

4. A supplier supplies 50 football scarves when the price is £6 each and 90 units when the price is £11
each.
a. Find the equation of supply function.
b. How many additional scarves are supplied for each successive £1 increase in price?
c. Find the quantity supplied when the price is £10 per scarf.
d. Find the price when 100 scarves are supplied.
5. If fixed cost of 20 articles is Rs. 500 and variable cost for each additional article is Rs. 40. Find the
total cost function. Also find the cost of 200 articles.
6. A company finds that the production cost associated with each articles is Rs. 40 and the fixed cost is
Rs. 30,000. If each article is sold for Rs. 60, find the following:
a. Total cost function
b. Profit function
c. Break-even point
d. How many articles should be produced to make a profit of Rs. 50,000?
7. A firm sells a single product for Rs. 70 per unit. Variable cost per unit are Rs. 20 for materials and
Rs. 30 for labour. Annual fixed cost is Rs. 100,000.
a. Construct the profit function in terms of x i.e. the number of units produced and sold.
b. What profit is earned if annual sales are 10,000 units?
8. The fixed cost of a new product is Rs. 400 and the variable cost per unit is 15. If the product can be
sold for Rs. 25 per unit, find the minimum number of items that should be produced to attains break-
even.
9. Asmita Book Publication finds that the production costs to each book are Rs. 50 and that the fixed
costs are Rs. 30,000. If each book can be sold for Rs. 90, determine:
a. The cost function
b. The profit function
c. The break-even point.
10. A firm sells a product for Rs. 55 per unit. The variable costs for material and labour are Rs. 20 and
Rs. 27 per unit. Find the profit function if the annual fixed cost is Rs. 100,000. Also, calculate the
profit earned from an annual sales of 15000 units.
15
11. If the average cost function is AC = 2Q – Q and demand function is P = 2 + Q, find the cost
function, revenue function, break-even quantity and break-even price.

ANSWER

1. a. 40 b. 20
2. a = 160, b = 2, P = 160 – 2Q
3. P = 55 – 0.5Q, Intercept at Q–axis is at Q = 110, Intercept at P–axis is at P = 55, P = $ 30
4. a. P = 0.125Q – 0.25 b. 8 c. 82
d. £ 12.25

5. TC = {500
500 + 40(Q – 20)
if Q ≤ 20
if Q > 20
, Rs. 7,700

6. (a) 40x + 30,000 (b) 20x – 30,000 (c) 1500 units


(d) 4,000 units
7. (a) 20x – 100,000 (b) Rs. 100,000
30 BUSINESS MATHEMATICS I

8. 40 units
9. (a) 50x + 30,000 (b) 40x – 30,000 (c) 750
10. 8x – 100,000, Rs. 20,000

11. (2Q – 15Q) × Q, 2Q + Q , 5 units and Rs. 7


2

1.13 Elasticity of Demand, Supply and Income


Elasticity of Demand (εd)
Elasticity is defined as the ratio of percentage change (proportionate change) in quantity demanded to the
percentage change (proportionate change) in other factors. For instance: price, income, tastes, preference,
etc. which are in short called determinants of demand.
Percentage change (or Proportionate change) in quantity demanded
Elasticity of Demand = Percentage change (or Proportionate change) in determinants of demand

There are three types of elasticity of demand.


1. Price elasticity of demand (εd)
2. Price elasticity of supply (εs)
3. Income elasticity of demand (εY)
Price Elasticity of Demand (εd)
Price elasticity of demand is defined as the change in demand in comparison to the change in price or the
ratio of percentage change (Proportionate change) in demand to the percentage change (Proportionate
change) in the price of a particular commodity. The price elasticity of demand (εd) usually yields a
negative value. We can also define εd as follows:
ΔQ
× 100
Percentage change in quantity demanded Q ΔQ P
εd = Percentage change in price = = × .
ΔP ΔP Q
P × 100

where
ΔQ = Change in quantity demanded
ΔP = Change in price
There are two ways of measuring price elasticity. They are explained as follows:
1. Point Elasticity of Demand
Let P = a – bQ be a linear demand function. Then the point elasticity of demand at any point (P0, Q0) is
defined by
ΔQ P0
εd = ×
ΔP Q0

or,
1 P0
εd = – b × Q ⎡∵ ΔQ = – 1⎤
0 ⎣ ΔP b⎦
NOTE Point elasticity of demand depends on price and vertical intercept only.
Linear Functions, Equations, Inequalities and Its Applications UNIT 1 31

2. Arc Price (Mid Points) Elasticity of Demand


The arc elasticity of demand measures the elasticity of demand over an interval on the demand function.
Arc elasticity uses the average of prices and quantities at the beginning and end of the given interval.
The formula for arc-price elasticity is
1
(P + P2)
ΔQ 2 1 ΔQ P1 + P2
εd = × = × .
ΔP 1 ΔP Q1 + Q2
2 (Q 1 + Q2)

For linear functions the arc price elasticity of demand can also be written as:
ΔQ P1 + P2 1 P 1 + P2 ⎡∵ ΔQ = – 1⎤
εd = × =–b×Q +Q .
ΔP Q1 + Q2 1 2 ⎣ ΔP b⎦

Example 1. If the price of a commodity increases by 10% and quantity demand decreases by 10%, find
price elasticity of demand.
Solution
Percentage change in demand = –10%
Percentage change in price = 10%,
Percentage change in demand –10%
εd = Percentage change in price = 10% = –1

Though real value of εd is negative, but economist ignore the sign and use positive value.
There are five types of elasticity of demand. They are as follows:
1. Perfectly elastic demand: The εd having its value ∞ is called perfectly elastic demand. Therefore,
εd = ∞.
2. Perfectly inelastic demand: The εd having its value 0 is called perfectly inelasticity demand.
Therefore, εd = 0.
3. Relatively elastic demand: The εd having its value greater than unity is called relatively elastic
demand. Therefore, εd > 1.
4. Relatively inelastic demand: The εd having its value smaller than unity is called relatively inelastic
demand. Therefore, εd < 1.
5. Unitary elastic demand: The εd having its numerical value equal to unity is called unitary elastic
demand. Therefore, εd = 1.
Example 2. Mention the type of elasticity under the given values of price and demand:
Price (Rs. per unit) Quantity Demanded (units)
10 100
8 120
Solution
We have
P1 = 10 Q1 = 100
P2 = 8 Q2 = 120
Change in price = ΔP = P2 – P1 = 8 – 10 = –2
Change in quantity demanded = ΔQ = Q2 – Q2 = 120 – 100 = 20
32 BUSINESS MATHEMATICS I

We know that
ΔQ P 20 10
εd = × = ×
ΔP Q –2 100
= –1
i.e. εd = 1 (Absolute value)
It is unitary elasticity of demand.

Price Elasticity of Supply (εs)


The economists have given idea about elasticity of supply in a similar way as they had done for demand
elasticity.
The price elasticity of supply (εs) is defined as the ratio of percentage change in quantity supplied to the
percentage change in price.
∆QS
× 100
Percentage change in quantity supplied QS ∆QS P
εs = Percentage change in price = ∆P = ∆P × Q .
S
P ×100
Types of price elasticity of supply are as follows:
1. Unitary elastic supply: If the elasticity is exactly one then εs is unitary elastic supply i.e. εs = 1.
2. Inelastic supply: If the elasticity is less than one then εs is inelastic supply, i.e. εs < 1.
3. Elastic supply: If the elasticity is greater than one then εs is elastic supply, i.e. εs > 1.
4. Zero elastic supply: If the quantity supplied does not make any change to the price then it is called
zero elastic supply, i.e. εs = 0.

Example 3. Find εs and hence, state its type by the values given below:
Percentage change in quantity supplied = 60%
Percentage change in price = 80%.
Solution
Percentage change in quantity supplied = 60%
Percentage change in price = 80%
We have
Percentage change in quantity supplied 60
εs = Percentage change in price = 80 = 0.75 < 1

Example 4. Calculate εs from the data given below:


Price (Rs. per unit) Quantity Supplied (units)
500 40
800 60
Solution
We have
P1 = 500 Qs1 = 40
P2 = 800 Qs2 = 60
Change in price = ΔP = 800 – 500 = 300
Linear Functions, Equations, Inequalities and Its Applications UNIT 1 33

Change in quantity supplied = ΔQs = 60 – 40 = 20


We know that
∆QS P
εs = ∆P × Q
S
20 500
= 300 × 40
5
= 6 < 1.
There are two types of price elasticity of supply.
1. Point Elasticity of Supply
Let P = c + dQ be a supply function. The point elasticity of supply at any point (P0, Q0) is defined by
ΔQ P0
εs = ×
ΔP Q0
1 P0
or, εs = d × Q .
0

Point elasticity of supply for linear function P = c + d Q depends on price and vertical intercept only.
2. Arc Price Elasticity of Supply
The arc price elasticity of supply is defined by
Q 2 – Q 1 P 1 + P2
εS = P – P × Q + Q .
2 1 1 2

For a linear supply function P = c + dQ, the arc price elasticity of supply is
ΔQ P1 + P2
εS = ×
ΔP Q1 + Q2
1 P 1 + P2
=d×Q +Q . ⎡∵ ΔQ = 1⎤
1 2 ⎣ ΔP d⎦
NOTE 1. If εs > 1, supply is elastic.
2. If εs < 1, supply is inelastic.
3. If εs = 1, supply is unit elastic.

Income Elasticity of Demand (εY)


The ratio of the percentage change (or proportionate change) in the quantity demand to the percentage
change (or proportionate change) in the income is called elasticity of income. It is also called income
elasticity of demand. Increase in income is accomplished by increase in quantity demanded.
ΔQ
Percentage change in quantity demanded Q × 100 ΔQ Y
εY = Percentage change in income = = = ×
ΔY ΔY Q
Y × 100

where Y and Q are initial income and initial quantity.


Example 5. Find income elasticity when the quantity demand for a good is increased by 10% resulting
in the increase of income by 8%.
34 BUSINESS MATHEMATICS I

Solution
Percentage change in quantity demand = 10
Percentage change in income = 8
Percentage change in quantity demanded 10
εY = Percentage change in income = 8 = 1.25.

Example 6. Calculate income elasticity of demand from the data given below
Income (Rs.) Quantity Demanded (units)
50,000 800
60,000 900
Solution
We have
Y1 = 50,000 Q1 = 800
Y2 = 60,000 Q2 = 900
Change in income = ΔY = 60,000 – 50,000 = 10,000
Change in quantity demanded = ΔQ = 900 – 800 = 100
We know that
ΔQ Y1 100 50‚000 5
εY = × = × 800 = 8 .
ΔY Q1 10‚000

1.14 Budget and Cost Constraints


Budget Constraint
A budget constraint refers to all the combination of goods and services that a consumer can purchase with
his/her income at given prices. The consumer has a limited income to purchase different goods and each
type of goods has a specified price per unit.
Consider two goods X and Y that a consumer wants to buy with his budget B. Let PX be the price per unit
of good X and PY be the price per unit of good Y. Let x and y be the quantities of goods X and Y
purchased by a consumer. Then the total budget is (x ⋅ PX + y ⋅ PY).
Thus, the equation of budget line is
x ⋅ PX + y ⋅ PY = B . . . (i)
Two determinants of the budget lines are
i. the prices of goods and
ii. the consumer's income to be spent on the goods.
The consumer can buy any combinations that lies on the budget line with his/her given income and given
prices of goods.
B
When x = 0, from (i) y = P .
Y
Linear Functions, Equations, Inequalities and Its Applications UNIT 1 35

B
This means that the maximum number of good Y that can be purchased is P when no good X is
Y
purchased. Y
B
Again, when y = 0, from (i) x = P . B
X
PY
This means that the maximum number of good X
B

Good Y
Budget line equation:
that can be purchased is P when no good Y is
X x ⋅ PX + y ⋅ PY = B
purchased.
PX
Slope of budget line (i) is – P .
Y

Since the increase in the number of one good X


O B
decreases the number of other good and vice-versa, PX
so the slope of budget line is negative.
Good X

Cost Constraints
The cost constraints or isocost line is a graphical representation of various combinations of two factors
labour and capital which the company or firm can afford or purchase with a given amount of money. The
isocost means equal cost line. That is, the iscost line shows all inputs with same total cost. Let w be the
wage rate of labour, r be the rental rate of capital, K be the amount of capital used, L be the amount of
labour used and C be the total cost of production then the equation of cost constraints is
wL + rK = C . . . (i)
If r = 0, equation (i) becomes Y

wL = C C
C r Isocost line (Line of cost constraint:
L=w. wL + rK = C
Capital (C)

C
Thus, w is the intercept on the axis in which labour
C
is shown. This means that w is the maximum amount
of labour when no capital is used.
If w = 0, equation (i) becomes X
O C
rK = C w
C
K= r . Labour (L)

C C
Thus, r is the intercept on the axis in which capital is shown. This means that r is the maximum amount
of capital when no amount in labour is used.
w
Slope of equation (i) = – r .

This negative slope indicates that increase in amount of capital decrease the amount of labour and vice-
versa.
36 BUSINESS MATHEMATICS I

Example 7. A consumer has an income of Rs. 2,000 to spend on the two goods X and Y whose prices
are Rs. 200 and Rs. 100 each respectively. What is the slope of budget constraints?
Solution
Price unit price of good X (PX) = Rs. 200
Price unit price of good Y (PY) = Rs. 100
We have
PX 200
Slope of budget constraints = – P = – 100 = –2.
Y

Example 8. A consumer has a budget of Rs. 3,000 to spend on two goods X and Y whose price per
units are Rs. 200 and Rs. 250 respectively.
a. Find the equation of budget line and sketch its graph.
b. If 10 units of good X are purchased then find the number of good Y that can be
purchased from the budget.
c. Show by calculation and graphically how the budget constraint changes
i. When the per unit price of good X increases to Rs. 300.
ii. When the per unit price of good Y decreases to Rs. 150.
iii. When the total budget increases to Rs. 4,000, the other variables do not change.
Solution
a. Price per unit of good X (PX) = Rs. 200
Price per unit of good Y (PY) = Rs. 250
Let x and y be the number of unit of goods X and Y respectively.
Total budget (B) = Rs. 3,000 Y
The equation of budget line is 15
x ⋅ PX + y ⋅ PY = B
12
or, x × 200 + y × 250 = 3,000
Good Y

9
∴ 4x + 5y = 60 …(i) 4x + 5y = 60
6
Table
3
x 0 15
y 12 0 X
O 3 6 9 12 15 18
Good X
b. When x = 10, from (i)
4 × 10 + 5y = 60
5y = 20
y=4
∴ Required number of good Y that can be purchased = 4 units.
Linear Functions, Equations, Inequalities and Its Applications UNIT 1 37

c.
Y
i. Now, price per unit of good X (PX) = Rs. 300
The new equation of budget line is 15

x ⋅ PX + y ⋅ PY = B 12
Original budget line

Good Y
or, x × 300 + y × 250 = 3,000 9
New budget line
∴ 6x + 5y = 60 …(i) 6
Table 3
x 0 10 X
O 3 6 9 12 15 18
y 12 0
Good X

When the price per unit of good X increases, then fewer units of X can be purchased.
This will cause the X-intercept to decrease towards origin but the vertical intercept (y-
intercept) will remain unchanged.
ii. Now, price per unit of good Y (PY) = Rs. 150 Y
The new equation of budget line is
24
x ⋅ PX + y ⋅ PY = B
21
or, x × 200 + y × 150 = 3,000
18
∴ 4x + 3y = 60 …(i) New budget line
Good Y

15
Table
Original budget line
12
x 0 15
y 20 0 9

When the price per unit of good Y decreases, 6


so more units of good Y can be purchased. 3
This will cause the y-intercept to increase
X
away from the origin but the horizontal O 3 6 9 12 15 18
intercept (x-intercept) will remain unchanged. Good X
iii. New budget = Rs. 4,000
The new equation of budget line is
x ⋅ PX + y ⋅ PY = B Y

or, x × 200 + y × 250 = 4,000 21


∴ 4x + 5y = 80 …(i) 18
Table 15 New budget line
x 0 20
Good Y

12
y 16 0 Original budget line
9
When the budget increases to
6
Rs. 40,000, so more of goods
can be purchased. 3

The new budget line rises and X


O 3 6 9 12 15 18 21 24
is parallel to the original budget
Good X
line. Both the lines have same
slope in this case.
38 BUSINESS MATHEMATICS I

Example 9. A firm wishes to spend Rs. 10,000 on a combination of two factors labour and capital for
producing a given level of output. If price of one unit of labour is Rs. 500 per day and the
price of capital is Rs. 1,000 per unit. Find the equation of isocost line. Also, find the isocost
line if the firm increases the total outlay to Rs. 15,000 and there is no change in the market
prices of the two factors labour and capital.
Solution
Price of one unit of labour (w) = Rs. 500
Price of capital per unit (r) = Rs. 1,000
Total cost (C) = Rs. 10,000
The equation of isocost is
wL + rK = C
or, 500L + 1000K = 10,000
or, L + 2K = 20 …(i)
If C = 15,000 then isocost line is
500L + 1000K = 15,000
L + 2K = 30 …(ii)
Equation (i) and equation (ii) are required equations of isocost lines.

 Exercise 1.5
1. If price elasticity of demand of a good is –0.5, then find the percentage change in demand when
a. the price increases by 4%.
b. the price decreases by 10%.
2. a. Given the demand function P = 10 – 0.2Q, calculate the point elasticity of demand at P = 5.
b. If the demand function is P = 1,200 – 1.5Q, determine the price elasticity of demand when
P = 900.
3. The demand function Q = 2,000 – 4P, where Q is the number of mobile toys demanded at Rs. P
each.
a. Derive an expression for the point elasticity of demand in terms of P only.
b. Calculate the point elasticity of each of the following prices, P = 100, 300, 400.
4. Calculate price elasticity of demand from following and explain the type of elasticity based on the
result.
a. The price of a commodity increase by 5%, and quantity demanded decreases by 8%.
b. Price (Rs./ Unit) Quantity demanded (Units)
15 10
20 4

c. Price (Rs./ Unit) Quantity demanded (Units)


20 6
30 4
5. The demand function of a good is given by P = 100 – 0.1Q. Find the arc price elasticity of demand
when
a. price decreases from Rs. 50 to Rs. 40.
b. price increases from Rs. 10 to Rs. 20.
Linear Functions, Equations, Inequalities and Its Applications UNIT 1 39

6. Calculate price elasticity of supply from the data given below:


a. Price (Rs./ Unit) Quantity Supplied (Units)
100 40
150 30

b. Price (Rs./ Unit) Quantity Supplied (Units)


100 40
160 50

c. Price (Rs./ Unit) Quantity Supplied (Units)


100 50
160 40
7. The supply function of a good is given by P = 10 + 0.2Q.
a. Find the point elasticity of supply when P = Rs. 20. Also find the percentage change in quantity
supplied in response to a price increase of 10% when P = Rs. 20.
b. Find the arc price elasticity of supply when price increases from Rs. 20 to Rs. 30.
8. a. The demand function of income is given by P = –75 + 0.5I. Determine the income elasticity of
demand at I = 200.
b. The demand P as a function of income I is given by 30 P = 10 + 2I. Find the expression for the
income elasticity of demand in terms of I and its value when I = 100.
9. Calculate the missing values.
a. Budget: B = ?, PX = 100, PY = 150, QX = 20, QY = 40
b. Budget: B = 10,000, P1 = 50, P2 = ?, Q1 = 100, Q2 = 250
c. Budget: B = 9,000, PX = 200, PY = 100, QX = 20, QY = ?
10. a. Ram has an income of Rs. 5,000 to spend on the two goods X and Y, whose prices arc Rs. 50
and Rs. 100 each respectively. What is the slope of budget constraint?
b. A consumer has an income of Rs. 1,600 to spend on the two goods X and Y whose prices are
Rs. 200 and Rs. 50 each respectively. What is the slope of budget constraints?
11. A person has a budget of Rs. 100 to spend on two goods X and Y whose prices are Rs. 5 and Rs. 10
on each, respectively.
a. Write the equation of budget line.
b. Find the slope and intercepts made by the budget line.
c. Sketch the graph of budget line.
d. Can the person buy 10 units of X and 6 units of Y with the given budget?
12. Find the unknown values from the equation of cost constraints.
a. Daily output = 600, No. of labour (L) = 10; Labour cost (w) = 20; Capital for materials (K) =
20, then find the rate of interest of capital (r) = ?
b. No. of labour (L) = 14, Capital invested (K) = 6, Labour cost (w) = 8, Rental rate of capital (r)
= 25, total investment of budget (C) = ?
13. For the production cost of an article, the limitation of cost is Rs. 40,000. If labour cost per unit is Rs.
500 and per unit cost of capital is Rs. 800, then
a. find the slope of line of cost constraints.
b. find the equation of cost constraints and intercepts made by the line of cost constraint.
40 BUSINESS MATHEMATICS I

ANSWER

1. (a) Quantity demand decreases by 2% (b) Quantity demand increases by 5%


2. (a) –1 (b) –3
P
3. (a) P – 500 (b) – 0.25, – 1.5, – 4

4. (a) 1.6 > 1 (Relatively elastic) (b) 1.8 > 1 (Positively elastic demand)
2
(c) 3 < 1 (Inelastic demand)

9 3
5. (a) – 11 (b) – 17

5 1
6. (a) – 0.5 (b) 12 (c) – 3

5
7. (a) 2, 20% (b) 3

I 20
8. (a) 4 (b) I + 5 , 21

9. (a) Rs. 8,000 (b) Rs. 20 (c) 50 units


1
10. (a) –2 (b) –4

1
11. (a) x + 2y = 20 (b) –2 , x-interpret = 20, y-intercept = 10

(c) Y (d) No.


Price of Good Y

15

10
x + 2y = 20
5

X
O 5 10 15 20 25
Price of Good X

12. (a) 20 % (b) 262


5
13. (a) – 8 (b) 5L + 8K = 400, L-intercept = 80, K-intercept = 50

1.15 Linear Functions, Equations and Inequalities using MS Excel


Arithmetic Operators
Following are some basic arithmetic operators frequently used in MS Excel
Operator Description
+ Addition
_ Subtraction
* Multiplication
/ Division
^ Exponentiation
% Percent

Comparison Operators
Some operators used for comparison purpose in MS Excel are listed below.
Linear Functions, Equations, Inequalities and Its Applications UNIT 1 41

Operator Description
= Equal to
> Greater than
>= Greater than or equal to
< Less than
<= Less than or equal to
<> Not equal to
Note that every Excel formula starts with equal symbol (=). After the equal symbol, you can enter either
a calculation or function, and then click enter to execute it.
Some Other Common Operators:
Operator Description
=count(E1:E11) Number of observations of the cell E from E1 to E11
=sum(E1:E11) Sum of the observations of the cell E from E1 to E11
=average(E1:E11) Arithmetic mean of observations of the cell E from E1 to E11
=median((E1:E11) Median of observations of the cell E from E1 to E11
=mode(E1:E11) Most frequently occurred value or mode of the cell E from E1 to E11
Please note that you can keep your data in any convenient cell in MS excel.
1. The graph of y = x + 4 in MS-Excel.
Step 1: Open the excel in your computer
Step 2: Have a look at different tabs at the top of the Excel screen. Among them, you will
find ‘Data’ tab as shown in the following.
Step 3: Mark x and y in as shown below.
Step 4: Calculate the y values using formula = B4+4 and drag down.
Step 5: Select the data and go to 'Insert' and click in scatter with straight lines and
markers.
42 BUSINESS MATHEMATICS I

2. The graph of demand function P = 200 – 0.5Q, for 0 ≤ Q ≤ 400.

3. Find the point of intersection of y = 8 – 3x and y = 3x – 4.


System of Linear
Equations and Its
Applications

 Learning Outcomes
On completion of this unit, students will be able to:
 Solve system of linear equations.
 Solve economic problems involving systems of linear equations.
 Use excel for system of linear equations and its applications.

System of Linear Equations


2.1 Introduction
An equation of the form ax + by = c is called a linear equation, where x, y are variables and a, b, c are
constants.
A system of linear equations is a collection of two or more linear equations involving the same variables.
For example: a1x + b1y = c1
a2x + b2y = c2 is a system of linear equations in two variables x and y.
The system of linear equations in three variables x1, x2 and x3 are
a11x1 + a12x2 + a13x3 = b1
a21x1 + a22x2 + a23x3 = b2
a31x1 + a32x2 + a33x3 = b3
where the coefficients of the variables and b1, b2, b3 are real constants.
The system of m linear equations in n variables (unknowns) is
a11x1 + a12x2 + . . . + a1nxn = b1
a21x1 + a22x2 + . . . + a2nxn = b2
– – – –
am1x1 + am2x2 + . . . + amnxn = bm
If bi = 0 for all i = 1, 2, . . ., m then the system of equations is called homogeneous system of equations.
A solution of the above system is a set (s1, s2, . . ., sn) of a numbers that makes each equation a true
statement when the values s1, s2, . . ., sn are substituted for x1, x2, . . ., xn.
The set of all possible solutions is called the solution set of the linear system.
A system of linear equations has either
1. no solution, or
44 BUSINESS MATHEMATICS I

2. exactly one solution, or


3. infinitely many solutions.
Definition
A system of linear equations is said to be consistent if it has either one solution or infinitely many
solutions and system is said to be inconsistent if it has no solution.
The system of equations a1x + b1y = c1 and a2x + b2y = c2 has
a1 b1
i. unique solution if a ≠ b .
2 2
a1 b1 c1
ii. infinitely many solutions if a = b = c .
2 2 2
a1 b1 c1
ii. no solution if a = b ≠ c .
2 2 2
For example: The system of equations x1 + x2 = 3 and x1 – x2 = 1 is consistent since it has a solution
namely x1 = 2 and x1 = 1.
But the system of equation x1 + x2 = 2 and 2x1 + 2x2 = 5 is inconsistent since it has no solution.
Three different cases are discussed in the following examples:
Y
Example 1: If the system has exactly one solution, the system is
said to be consistent and independent. For example, 4
3
consider equations 3x + 2y = 8 and –x + 3y = 1. The solution
2
of this system is x = 2, y = 1. The lines represented by these 1 (2, 1)
equations are intersecting and they intersect at (2, 1). Such X
-3 -2 -1 0 1 2 3 4 5
system is called consistent and independent. The graph is
shown in the adjoining figure. 3x + 2y = 8
–x + 3y = 1
Y
Example 2: If the system has an infinite number of solutions, the
system is said to be consistent and dependent. Consider 4
3
x + y = 2 and 2x + 2y = 4. Here two equations describe the
2
same line. Both lines have infinite common solutions. The 1
solution for this system is the set of all points on the line. The X
-3 -2 -1 0 1 2 3 4 5
graph is shown in the adjoining figure. x+y=2
2x + 2y = 4
Y
Example 3: If the system has no solution, the system is said to be
inconsistent and independent. Consider x + y = 2 and 5
x + y = 5. From the graph, we see that the two lines do not 4
3
intersect since the lines are parallel. Thus, the system has no
2
solution. The graph is shown in the adjoining figure. 1
X
-3 -2 -1 0 1 2 3 4 5
x+y=2 x+y=5

Trivial and Non-trivial Solution


If all the values of the variables in a system of equations is zero then the solution is called trivial solution.
If at least one value of the variable is non-zero, then the solution is called non-trivial solution.
There are several techniques, for solving such intersecting lines. We shall illustrate some of them with the
help of examples.
System of Linear Equations, and Its Applications UNIT 2 45

Example 1. Solve the following linear equations by (a) Substitution Method, (b) Elimination method
and (c) Cross multiplication method.
x + 3y = 4 and 3x – 2y = 1.
Solution
a. Substitution Method b. Elimination method
x + 3y = 4 . . . (i) Given equations are
3x – 2y = 1 . . . (ii) x + 3y = 4 . . . (i)
From equation (i) 3x – 2y = 1 . . . (ii)
x + 3y = 4 Multiplying equation (i) by 2 and
or, 3y = 4 – x equation (ii) by 3 and adding
4–x 2x + 6y = 8
∴ y= 3 . . . (iii) 9x – 6y = 3
Substituting the value of y in equation 11x = 11
(ii), we get ∴ x=1
4 – x
3x – 2⎛ 3 ⎞ = 1
Substituting the value of x in
⎝ ⎠ equation (i), we get
9x – 8 + 2x 1 + 3y = 4
or, 3 =1
or, 3y = 4 – 1
or, 11x – 8 = 3 or, 3y = 3
or, 11x = 3 + 8 ∴ y=1
or, 11x = 11 Therefore, solution the equation is
∴ x=1 (x, y) = (1, 1).
Substituting the value of x in equation
(iii), we get
4–1 3
y= 3 =3=1
Hence, required solution is (x, y) = (1, 1).
c. Cross multiplication method
Given equations are
x + 3y = 4
∴ x + 3y – 4 = 0 . . . (i)
3x – 2y = 1
∴ 3x – 2y – 1 = 0 . . . (ii)
Solving by cross multiplication method
x y 1
3(–1) – (–2) (–4) = (–4) ⋅ 3 – (–1) 1 = 1(–2) – 3 ⋅ 3
x y 1
or, –3 – 8 = –12 + 1 = –2 – 9
x y 1
or, –11 = –11 = –11
Taking 1st and 3rd expressions Taking 2nd and 3rd expressions
x 1 y 1
–11 = –11 –11 = –11
–11 –11
or, x = –11 or, y = –11
∴ x=1 ∴ y=1
Therefore, the solution of above equation are (1, 1).
46 BUSINESS MATHEMATICS I

Example 2. Find the point of intersection of y = 8 – 3x and y = 3x – 4.


Solution Y
A
Here, y = 8 – 3x
9
x 0 2 D
8 (0, 8)
y 8 2
7 y = 8 – 3x
The straight line AB denotes y = 8 – 3x 6
y = 3x – 4

Again, y = 3x – 4 5 (3, 5)
x 0 3 4

y –4 5 3
2 (2, 2)
The straight line CD denotes y = 3x – 4.
1
From the figure of the graph, point of
X' X
intersect of AB = y = 8 – 3x and y = 3x – 4 O
-1 1 2 3 4 5 6
is the point P(2, 2). i.e. (x, y) = (2, 2). -2
Note that the point of intersection of the -3
B
lines also can be calculated by solving the -4 (0, –4)
given equations. -5

C Y'

2.2 System of Linear Equations with Three Variables


System of linear equations with three variables are more complicated in comparison to those of two
variables while solving. We can use elimination method or substitution method to solve such types of
equations. There are other various methods to solve these equations.
Example 3. Solve: 2x – y + z = –1, x – 2y + 3z = 4 and 4x + y + 2z = 4.
Solution
We have the equations:
2x – y + z = –1 . . . (i)
x – 2y + 3z = 4 . . . (ii)
4x + y + 2z = 4 . . . (iii)
Take equation (i) to eliminate z,
z = –2x + y – 1 . . . (iv)
Substituting z in equations (ii) and (iii) respectively
x – 2y + 3(–2x + y – 1) = 4
or, x – 2y – 6x + 3y – 3 = 4
or, –5x + y = 4 + 3
∴ –5x + y = 7 . . . (v)
And
4x + y + 2(–2x + y – 1) = 4
or, 4x + y – 4x + 2y – 2 = 4
or, 3y = 4 + 2
or, 3y = 6
∴ y=2
Substituting y = 2 in equation (v), we get
–5x + 2 = 7
or, 5x = 7 – 2
or, –5x = 5
System of Linear Equations, and Its Applications UNIT 2 47

∴ x = –1
Substituting the value of x = –1 and y = 2 in equation (iv), we get
z = –2 (–1) + 2 – 1
or, z = 2 + 2 –1
∴ z=3
Therefore, x = –1, y = 2, and z = 3.
Example 4. Solve: x + 2y + 3z = 6, 2x + 4y + z = 7, and 3x + 2y + 9z = 14.
Solution
Given
x + 2y + 3z = 6 . . . (i)
2x + 4y + z = 7 . . . (ii)
3x + 2y + 9z = 14 . . . (iii)
Multiplying equation (i) by 2 and subtracting Again, multiplying equation (i) by 3 and
equation (ii) from equation (i) subtracting equation (iii) from equation (i)
2x + 4y + 6z = 12 3x + 6y + 9z = 18
2x + 4y + z = 7 3x + 2y + 9z = 14
– – – – – – – –
5z = 5 4y = 4
∴ z=1 ∴ y=1
Substituting the values of x and y in equation (i),
x+2×1+3×1=6
or, x + 5 = 6
or, x = 6 – 5 = 1
∴ x = 1, y = 1, z = 1

 Exercise 2.1
1. Test whether the given linear equations have no solution, unique solution or many solutions.
a. 2x + 3y = 2 and 5x + 2y = 6 b. 4x – 6y = 8 and 3y – 2x = – 4
c. y – x = 2 and y = 3 + x.
2. Solve the following system of equations.
a. x + y = 6 and 3x – y = 2 b. x – 2y = 4 and 2x – 4y = 9
c. 3x – 2y = 8 and 5x + 3y = 7 d. 5x – 3y = 20 and 2x + 3y = 8
x y 2 5 5
e. 2 + 5 = 5 and x + y = 6 .

3. Solve the following system of linear equations:


a. x + 2y + z = 0, 2x + y – z = 0 and x + y + 2z = 0
b. 2x – y + 3z = 9, x+y+z=6 and x – y + z = 2
c. 2x + 2y + z = 16, x + 2y = 10 and 2y + 3z = 12
d. x + y + z = 9, 2x + 5y + 7z = 52 and 2x + y – z = 0.
4. 40 people are employed in a factory. If the daily total wage bill of the factory is Rs. 3625 and the
daily wage of a man and a women are Rs. 100 and Rs. 75 respectively. Find the number of men and
women employed in the factory.
5. An aeroplane has 150 seats for passengers. Every passenger willing to pay business class fare is
allowed to carry 30 kgs of baggage. But economic class passengers are restricted to carry only 20
48 BUSINESS MATHEMATICS I

Kgs of baggage each. If only 4000 Kgs. of baggage can be carried, find the number of passengers of
each kind.
6. There are two commodities X and Y. Mr. A purchases 1 unit of X and sells 3 units of Y. Mr. B
purchases 3 units of Y and sells 2 units of X. In this process Mr. A and Mr. B get Rs. 500 and 200
respectively. Find the price per unit of each commodity.
7. The three commodities C1, C2 and C3 are purchased and sold by the three persons A, B and C. Mr. A
purchases 4 units of C3 and sells 3 units of C1 and 5 units of C2. Mr. B purchases 3 units of C1 and
sells 2 units of C2 and 1 units of C3. Mr. C purchases 1 unit of C2 and sells 4 units of C1 and 6 units
of C3. In the purchase - sell process Mr. A suffered a loss of Rs.1,000; Mr. B earns no profit and Mr.
C earns Rs.40,000. Find the prices per unit of these commodities.

ANSWERS
1. (a) Unique solution (b) Infinitely many solutions (c) No solution
2. (a) 2, 4 (b) No solution (c) 2, – 1 (d) 4, 0
(e) 4, 15
3. (a) 0, 0, 0 (b) 1, 2, 3 (c) 4, 3, 2 (d) 1, 3, 5
4. 25, 15 5. 100, 50 6. Rs. 700 and Rs. 400
7. Rs. 3,000, Rs. 2,000 and Rs. 5,000
System of Linear Equations, and Its Applications UNIT 2 49

Application of System of Linear Equations


In this section, we study about the application of system of linear equation in management and
economics.

2.3 Demand and Supply Analysis


Demand for a commodity is related to the price of commodity. The law of demand states that the quantity
demanded will increase as price decreases and that the quantity demanded will decreases as price
increases.
The law of supply states that the quantity supplied for sale will increase as the price of a commodity
increases.
Let Qd and Qs be the quantity demanded and quantity supplied of a good. Then
1. If Qd > Qs then there will be the excess of demand. The excess of demand is Qd – Qs. This leads to
the scarcity of good in the market.
2. If Qd < Qs then there will be the excess of supply. The excess of supply is Qs – Qd. This is the
surplus of good.
3. If Qd = Qs then this is the market equilibrium condition. The equilibrium point is that where the
demand and supply curves intersect.
Example 1. Assume that the demand function is linear in nature. The quantity demanded of a good is 55
units when the price is Rs. 10 and 30 units when the price is Rs. 15.
Determine the equation of the demand function in the form Q = f (P). Plot the graph of the
demand function.
Solution
The demand function is linear in nature. So, let the equation of demand function be
P = a – bQ . . . (i)
When P = Rs. 10, Q = 55
So, from (i)
10 = a – 55b . . . (ii)
Again, when P = Rs. 15, Q = 30
So, from (i)
15 = a – 30b . . . (iii)
Subtracting equation (ii) from equation (iii)
a – 30b = 15
a – 55b = 10
– + –
25b = 5
1
b = 5 = 0.2
50 BUSINESS MATHEMATICS I

Putting the value of b in equation (ii), we get


10 = a – 55 × 0.2
or, 10 = a – 11 Y
∴ a = 21
126
Putting the value of a and b in equation (i)
105
1

Quantity (Q)
P = 21 – 5 Q 84

Q 63 Q = 105 – 5P
or, 5 = 21 – P 42
∴ Q = 105 – 5P 21
Table
X
P 0 21 O 7 14 21 28
Q 105 0 Price (P)
The graph of demand function is shown along side.
Equilibrium in the Goods and Labour Market
1. Goods Market Equilibrium
When the quantity demanded (Qd) by consumers and the quantity supplied (Qs) by producers of a good or
service are equal then it is called goods market equilibrium or market equilibrium. In other words, market
equilibrium occurs when the price that a consumer is willing to pay (Pd) is equal to the price that a
producer is willing to accept (Ps).
Mathematically, the equilibrium condition can be expressed as
Qd = Qs and Pd = Ps.
The price and quantity in equilibrium condition are called equilibrium price and equilibrium quantity
respectively.
2. Labour Market Equilibrium
When the labour demanded (Ld) by firms is equal to the labour supplied (Ls) by workers then it is said to
be labour market equilibrium. In other words, when the wage that a firm is willing to offer (Ws) is equal
to the wage that workers are willing to accept (Wd), it is labour market equilibrium.
Mathematically, it can be expressed as Ld = Ls and Wd = Ws.
Example 2. Demand and supply functions for the products in two markets are given by:
Qd1 = 82 – 3P1 + P2 Qs1 = 15P1 – 5 and
Qd2 = 92 + 2P1 – 4P2 Qs2 = 32P2 – 6
Determine the price and quantity to both the markets in equilibrium market condition.
Solution
We have the following functions for demand and supply functions of the markets.
Qd1 = 82 – 3P1 + P2 . . . (i)
Qs1 = 15P1 – 5 . . . (ii)
Qd2 = 92 + 2P1 – 4P2 . . . (iii)
Qs2 = 32P2 – 6 . . . (iv)
As the market is in equilibrium, the quantity demanded is equal to the quantity supplied.
System of Linear Equations, and Its Applications UNIT 2 51

Qd1 = Qs1
or, 82 – 3P1 + P2 = 15P1 – 5
or, P2 = 15P1 – 5 – 82 + 3P1
∴ P2 = 18P1 – 87 . . . (v)
And
Qd2 = Qs2
or, 92 + 2P1 – 4P2 = 32P2 – 6
or, 92 + 6 + 2P1 = 32P2 + 4P2
or, 98 + 2P1 = 36P2
or, 98 + 2P1 = 36 (18P1 – 87) [From (v)]
or, 98 + 2P1 = 648P1 – 3132
or, 98 +3132 = 648P1 – 2P1
or, 3230 = 646P1
3230
or, P1 = 646

∴ P1 = 5
From (v) substituting P1 = 5, we get
P2 = 18 × 5 – 87
= 90 – 87
∴ P2 =3
Therefore, P1 = 5 and P2 = 3.

2.4 Break-Even Analysis


The point at which total costs and total revenues are equal is called break even point. The break even
point can be described as a point where profit is zero.
Mathematically, Y

Profit (π) = 0 Total Revenue (TR)


or, TR – TC = 0
∴ TR = TC Break Even Point
Cost and Revenue

Profit
(B.E.P.) Total Cost
(TC)

Variable Cost

Loss

Fixed Cost

X
O

Quantity
Example 3. A manufacturer sells a product for $10 per unit. The manufacturer's fixed cost are $1200
per month, and the variable costs are $2.50 per unit. Find the break even point both
algebraically and graphically.
52 BUSINESS MATHEMATICS I

Solution
Let x be the total units of the product.
Revenue function R(x) = 10x
Cost function C(x) = Fixed cost + Variable cost
= 1,200 + 2.50x
For break-even point,
R(x) = C(x)
or, 10x = 2.5x + 1,200
or, 7.5x = 1,200
∴ x = 160
The manufacturer will be in break even if 160 units are produced per month.
Y

2800
R = 10x
2400
Cost and Revenue

Break-even point
2000 Profit Region
C = 2.5x + 1200
1600
(160, 1600)
1200
Loss Region
800

400

X
O
40 80 120 160 200 240 280 320 360
Quantity (Q)

The above figure shows that for x < 160, R(x) < C(x) there is loss and for x > 160,
R(x) > C(x), there is profit.

2.5 Consumer and Producer Surplus


Consumer's Surplus
Y
The area between the equilibrium price and the demand curve is
actually consumer surplus.
B Demand Curve
In the price and quantity demand curve ΔABC is the consumer's Equilibrium
Price

surplus. point
In the figure alongside,
A
Payment the consumer wanted to make = OBCD C

Payment made by the consumer = OACD O


X
D
Then, Quantity

Consumers surplus = Payment s/he wanted to pay – Payment s/he made


= OBCD – OACD
1
= ΔABC = 2 × AC × AB.
System of Linear Equations, and Its Applications UNIT 2 53

Example 4. The demand and supply function of a product is QD = 30 – P and QS = 5P – 6. Find


consumer's surplus along with price and quantity under equilibrium condition.
Solution
Given
QD = 30 – P . . . (i)
QS = 5P – 6 . . . (ii)
For equilibrium condition
QD = QS
or, 30 – P = 5P – 6
or, 6P = 36
∴ P = 6.
Then,
QD = 30 – P = 30 – 6 = 24 = QS i.e. Equilibrium price = P = 6
and Equilibrium quantity = Q = 24 and point is (24, 6)
But when Q = 0; then P = 30 – 0 = 30 and point is (0, 30)
The graph is alongside consumer surplus Y

= Area of (OBCD – OACD) 32 B(0, 30)


= Area of ΔABC 28
1 24
= 2 × AC × AB 20 Demand Curve
Price

1 16
= 2 × OD × (OB – OA) 12
8 C(24, 6)
1 A
= 2 × 24 × (30 – 6) 4
X' X
= 288. O
-1 4 8 12 16 20 24 28 32

Producer's Surplus Y' Quantity

It is the difference of price between the sum received by a producer to the sum s/he was expecting.
In the price and quantity sold Δ ADC is the producer's surplus. In
the figure alongside. Y

Sum producer's receives = Fig. OACB


Sum producer's expected to receive = Fig. ODCB C
A
Producer surplus = Fig. OACB – Fig. ODCB
Price

= ΔADC
Supply Curve
1 D
= 2 × AD × AC.
X
O B
Single figure shown below explain both consumer as well as Quantity Sold
producer's surplus.
54 BUSINESS MATHEMATICS I

C
Demand Curve
Consumer's P
Surplus D
Price
B Equilibrium
Producer's point
Surplus
Q
Supply Curve
A

X
O
Quantity

Example 5. The demand and supply functions of a product are QD = 30 – P, and QS = 5P – 6. Find
producer's surplus.
Solution
We have seen in previous example
QD = QS, gives, P = 6 and
QD = QS = 24
(24, 6) is one of the point of supply curve. But when QS = 0, then
5P – 6 = 0
6 6
∴ P = 5 , we have another point is ⎛0‚ 5⎞ = (0, 1.2)
⎝ ⎠
From the figure below. Y

Producer's surplus = Area of (ODCA – OBCA) 10


= ΔBCD 8
D C(24, 6)
1
Price

6
= 2 × OA × (OD – OB)
4
1 2 Supply Curve
= 2 × 24 × (6 – 1.2) B A
X' X
O
= 12 × 4.8 -1 4 8 12 16 20 24 28 32
= 57.6 Y' Quantity

2.6 National Income Determination


National Income
The earning made by people of a country within one year from various productive resources (two or three
or n resources) is called National income.
Therefore,
National Income (NI) = Y1 + Y2 + Y3 + . . . + Yn
where Y1 + Y2 + Y3 + . . . + Yn are the individual incomes of n people of the country.
Following are the concepts of national income.
1. Gross Domestic Product (GDP): It is the total market value of all currently produced goods and
services in a certain year within any country. Therefore,
System of Linear Equations, and Its Applications UNIT 2 55

GDP = Consumption + Investment + Government spending + Net exports


2. Gross National Product (GNP): It is the sum of total market value of all currently produced goods
and services and net factor income from abroad (NFIA) in a certain year of any country. Therefore,
GNP = GDP + NFIA
3. Net National Product at Market Price (NNPMP): It is the net GNP value obtained by subtracting
the depreciated value of machines (or physical capital goods). Therefore,
NNPMP = GNP – Depreciation
4. National Income (NI) or Net National Product at Factor Cost (NNPFC): It is the net output of
market value of all currently produced goods and services in a certain year to the hands of the
ultimate consumers from the country's productive system. It is there, also called Net National
Product at Factor Cost (NNPFC). Therefore,
NI = NNPFC = NNPMP – Indirect taxes + Subsidies (if any)
5. Personal Income (PI): It is the total earnings of an individuals and households from all possible
sources in certain year before direct sources. Therefore,
PI = NI − Corporate income taxes – Undistributed corporate profit –Social security
contribution + Transfer payments (if any)
6. Disposable Income (DI): It is the net income left after deducting direct taxes from personal income.
Therefore,
DPI = PI – Direct taxes
7. Per Capita Income (PCI): It is the ratio of national income of a certain year to the total population
of the country. Therefore,
National income of a year
PCI = Total population of that year

The level of national income at which the production and purchasing plans of the economy are
synchronized is called the national income determination. It is the theory of national income concerned
with finding out the expenditure level of national income.
National income determination can be explained in a
1. Two sector economy (i.e. household consumption = C and business investment = I),
2. Three sector economy (i.e. household consumption = C, business investment = I and government
expenditure = G),
3. Four sector economy (i.e. household consumption = C, business investment = I, government
expenditure = G and extra expenditure = E)
Here, we take a two sector economy, from a graph of equilibrium output of national income (Y) with
household consumption (C) and business investment (I).
1. On the basis of graph of income and investment equilibrium
The linear equation of the income and investment is
Y=C+I . . . (i) and
C = Ca + bY . . . (ii) so that
Y = Ca + bY + I . . . (iii)
where
Y = National income
Ca = Autonomous consumption
I = Business investment
b = Marginal propensity of consumption (MPC) where a ≤ b ≤ 1.
56 BUSINESS MATHEMATICS I

Y=C+S

Saving and Investment


E Y=C+I

C = Ca + bY
I

C
45°
X
O Output/ Income (Y)

2. On the basis of saving investment equilibrium


In this case
Saving (S) = Investment (I)
or, I = –Sa + (1 – b)Y
where
1 – b = Marginal propensity to saving = s
Y
Saving and Investment

S = –Sa + (1 – b)Y

E1
1 I

X
O Ye

Output/ Income (Y)

Example 6. Suppose that equilibrium, Y = Rs. 650, MPC = b = 0.8, consumption = C = Rs. 60 + bY,
find business investment, I.
Solution
We know that
Y=C+I
or, Y = 60 + bY + I
or, (1 – b)Y = 60 + I
or, (1 – 0.8) × 650 = 60 + I
or, 0.2 × 650 – 60 = I
or, I = 130 – 60
∴ I = Rs. 70
System of Linear Equations, and Its Applications UNIT 2 57

Example 7. The saving function is S = –20 + 0.25Y where Y is national. If the autonomous investment,
I = Rs. 500, find Y.
Solution
We know for equilibrium of saving
S=I
or, –20 + 0.25Y = 500
or, 0.25Y = 500 + 20
or, 0.25Y = 520
520
∴ Y = 0.25 = Rs. 2080

Measurement of National Income


National income can be measured by the following three methods:
Product Method
This method measures national income from the total sum of the market value of all the final goods and
services produced in the economy.
1. Final Product Method: In this method, national income is estimated by finding the market value of
all the final goods and services produced in an economy in a year. The method of calculating
national income by this method is as follows:
GDP = Market value of all the final goods and services produced within the country
= P1 Q1 + P2 Q2 + P3 Q3 + . . . + Pn Qn
GNP = GDP + Net factor income from aboard
NNP = GNP – Depreciation
NNPFC = NNP – Net Indirect tax
NI = NNPFC
where
P = Price of the respective goods and services
Q = Quantity of goods and services
2. Value Added Method: In this method, the value added or created at different stages of the
production is counted for estimating national income. Thus,
Gross value added = Value of output – Cost of intermediate goods
GDP = Gross value added in various sectors within the country
GNP = GDP + Net factor income from abroad
NNP = GNP – Depreciation
NNPFC = NNP – Net Indirect tax
NI = NNPFC
Income Method
In this method, the incomes received by all the residents of a country for their productive services during
a year are added up to obtain national income.
The method of calculating NI by income method is as follows:
GDP = Wages/Salaries + Interest + Rent + Profit + Depreciation+ Net Indirect Taxes
GNP = GDP + Net factor income from abroad
58 BUSINESS MATHEMATICS I

NNP = GNP – Depreciation


NNPFC = NNP – Net Indirect tax
NI = NNPFC
Expenditure Method
In this method, the economy is divided into four major sectors: household, government, business, and
foreign. These are the major markets for the goods and services produced by an economy.
The calculation of national income by this method is as follows:
GDP = C + I + G + (X – M)
GNP = GDP + Net factor income from abroad
NNP = GNP – Depreciation
NNPFC = NNP – Net Indirect tax
NI = NNPFC
where
C = Private consumption expenditure
I = Private investment expenditure
G = Government expenditure
X = Export
M = Import
X – M = Net export
Example 8. Find the per capita income of Ecuador in 2016 AD when national income and population
were $195,100,000,000 and 16,490,000 respectively.
Solution
We have
National income in 2016 = $19,510,000,0000
Population of Ecuador in 2016 = 16,490,000
Then,
National income in 2016 $195‚100‚000‚000
PCI = Total population in 2016 = 16‚490‚000 = $11831.41

Example 9. Find the value of Y (a) National income equilibrium (b) Saving investment equilibrium,
Autonomous consumption = Ca = 20, MPC = b = 0.75, I = 20.
Solution
a. National income equilibrium b. Saving investment equilibrium
We know that Here,
Y=C+I S=I
or, S = –Sa+ sY, where s = 1 – b
or, Y = Ca + bY + I
or, I = –S + (1 – 0.75)Y
or, (1 – b)Y = Ca + I
or, 20 + 20 = 0.25Y
or, (1 – 0.75)Y = 20 + 20 or, 40 = 0.25Y
or, 0.25Y = 40 40
or, Y = 0.25 = 160
40
or, Y = 0.25 = 160.
∴ Y = 160.
∴ Y = 160.
System of Linear Equations, and Its Applications UNIT 2 59

 Exercise 2.2
1. Two points on a linear demand function are (Rs. 20, 80000) and (Rs. 30, 62500). Determine
a. Demand function Q = f (P).
b. Price when demand is of 50,000 units.
c. Slope of the function.
2. Assume that the demand function is linear in nature. The quantity demanded of a good is 55 units
when the price is Rs. 10 and 30 units when the price is Rs. 15.
a. Determine the equation of the demand function in the form Q = f (P). Plot the graph of the
demand function.
b. Use the equation of the demand function to find the change in demand when the price (i)
increases by Rs. 2 (ii) decreases by Rs. 3.
3. Find the equilibrium price and quantity:
a. Qd = 30 – P; Qs = –6 + 5P
8 2
b. Qd = 50 – 7 P; Qs = 10 + 3 P
c. Qd = 2 + 3P; Qs = 2P + 7
4. Given the following demand and supply functions for the competing products in two markets; for
market equilibrium condition find P1 and P2.
a. Qd1 = 10 – 2P1 + P2 and Qs1 = –2 + 3P1
Qd2 = 15 + 2P1 – P2 and Qs2 = –1 + 2P2
b. Qd1 = –14 – P1 + 2P2 and Qs1 = P1 – P2 – 2
Qd2 = 7 + 2P1 – P2 and Qs2 = 11 – 2P1
c. Qd1 = 12 + 2P1 – P2 and Qs1 = 19 + 3P1 – 5P2
Qd2 = 6 – P1 + P2 and Qs2 = 10 + P1 + 2P2
5. The ABC company finds that the production cost associated with each article is Rs. 40 and fixed
cost is Rs. 30,000. If each article is sold for Rs 60, find the followings.
a. Total cost function.
b. Profit function.
c. Break even point.
d. How many articles should be produced to move a profit of Rs. 50,000.
6. If the demand function for a product is P = 8 – 2x and the supply function is P = 2x where P is the
unit price in Rs and x is the quantity demanded (the quantity supplied in units), find the consumer's
surplus and the producer's surplus at the equilibrium price level.
7. The demand and supply functions for a good are given by
Demand function: P = 52 – 0.3Q
Supply function: P = –20 + 0.3Q
a. Calculate the equilibrium price and quantity.
b. Find the consumer surplus and procedure surplus.
c. Illustrate graphically.
8. The demand and supply function for a good are given by:
Pd = 58 – 0.2Q
Ps = 4 + 0.1Q
Calculate the consumer's surplus, producer's surplus and total surplus.
60 BUSINESS MATHEMATICS I

9. Find the equilibrium national income Y in the basic Keynesian macroeconomic model.
Y = C + I, C = 20 + 0.6 Y, I = 60.
10. For an economy, the consumption function is C = 60 + 0.75Y. If the investment in a year is Rs. 35
millions, what will be the equilibrium level of income?
11. In an economy which engages in foreign trade, it is assumed that
Y=C+I+G+X–M
C = 0.7 Yd
M = 0.15Yd
I = 300, G = 250, X = 120, T = 0.3
Determine the equilibrium level of national income and equilibrium level of consumption.

ANSWERS
1. (a) Q = 1, 15,000 – 1750P (b) 1137.14 (c) –1750
2. (a) Q = 105 – 5P (b) i. Quantity demanded is decreased by 10 units.
ii. Quantity demanded is increased by 15 units.
Y

126

105
Quantity (Q)

84

63 Q = 105 – 5P

42

21

X
O 7 14 21 28
Price (P)

3. (a) 6, 24 (b) 22.11, 24.74 (c) 5, 17


4. (a) 4, 8 (b) 0, 4 (c) 1, 2
5. (a) 40x + 30,000 (b) 20x – 30,000 (c) 1,500 units (d) 4,000 units
6. 4, 4
7. (a) 16, 120 (b) 2160, 1493.33
8. 3240, 1620, 4860
9. 200
10. 380 millions
11. 1089.43, 533.82
System of Linear Equations, and Its Applications UNIT 2 61

2.7 System of Linear Equations using MS Excel


1. Test whether the given linear equations have no solution, unique solution or many solutions:
y = x + 2 and y = 3 + x.

From graph we see that the system has no solution.


2. Solve: y = 6 – x and y = 3x – 2


62 BUSINESS MATHEMATICS I
Quadratic Functions
and Equations and Its
Applications
 Learning Outcomes
On completion of this unit, the students will be able to:
 Solve quadratic equations, quadratic inequalities.
 Sketch the graph of a quadratic function with its main characteristics.
 Solve economic and management problems involving quadratic equations.
 Use excel for quadratic functions and equations and its applications.

Quadratic Functions and Equations


3.1 Quadratic Functions
A function f(x) defined by
f(x) = a0xn + a1xn-1 + a2xn-2 + a3xn-3 + ⋅⋅⋅ + an-1x + an , (a0 ≠ 0)
is said to be a polynomial of degree n in x, where a0, a1, a2, ⋅⋅⋅, an ∈ § are all constants, n is a non-
negative integer and a0xn, a1xn-1, ⋅⋅⋅, an are the terms of the polynomial. The term a0 xn is called its leading
term and the constants a0, a1, a2, ⋅⋅⋅, an are called the coefficients of the polynomial.
If n = 2 then polynomial becomes quadratic function. Now we define quadratic function.
A function of the form f (x) = ax2 + bx + c, (a ≠ 0), is called a quadratic function, a, b, c being the
arbitrary constants. In other words, a polynomial function having highest power of the variable 2, is called
a quadratic function; and is expressed as f (x) = ax2 + bx + c. The term ax2 is called leading term and a is
called leading coefficient. For example: y = 2x2 + 3x + 5, y = x2, y = –3x2 + 5, y = 2x2 – 4, etc.
The graph of quadratic function gives a parabola as shown in the Y

figure alongside.
P(x, y)
Parabola is the locus of a point in a plane which are taken in such a M

way that they are equidistant from a fixed point (called focus) and a
fixed straight line (called directix). In figure, PF = PM where F is the X' X
Z O A F
focus and ZM is the directrix, and XOX' is the axis of parabola (or
axis of symmetry). Also A lying at origin O(0, 0) is called vertex.

Y'
64 BUSINESS MATHEMATICS I

Let us take a quadratic function


f (x) = ax2 + bx + c
b c
= a ⎛x2 + a ⋅ x + a⎞
⎝ ⎠
b b 2 b 2 c
= a ⎡x + 2 ⋅ x ⋅ 2a + ⎛2a⎞ – ⎛2a⎞ + a⎤
2
⎣ ⎝ ⎠ ⎝ ⎠ ⎦
2 2
b b c
= a ⎡⎛x + 2a⎞ – 4a2 + a⎤
⎣⎝ ⎠ ⎦
2 2
b b – 4ac
= a ⎡⎛x + 2a⎞ – ⎛ 4a2 ⎞⎤
⎣⎝ ⎠ ⎝ ⎠⎦
2 2
b 4ac – b
∴ f (x) = a ⎛x + 2a⎞ + 4a
⎝ ⎠
b b 4ac – b2
Put x = – 2a then, f ⎛–2a⎞ = 4a
⎝ ⎠
b 4ac – b2
Therefore, the vertex of parabola = ⎛–2a‚ 4a ⎞ .
⎝ ⎠
b 4ac – b2
Thus, the graph of the function y = ax2 + bx + c, a ≠ 0 is a parabola having vertex at ⎛–2a‚ 4a ⎞ . The
⎝ ⎠
parabola opens up (concave upward) if a > 0 and opens down (concave downward) if a < 0.
If a > 0 then the vertex is at minimum point and if a < 0 then the vertex is at maximum point. The sketch
of the graph of parabola y = ax2 + bx + c is with a < 0 and a > 0 are given below.

Vertex
(Maximum Point)
a>0

a<0
Vertex
(Minimum Point)

axis axis
(Parabola turning upward)
(Parabola turning downward)
Quadratic Functions and Equations and Its Applications UNIT 3 65

3.2 Sketching the Graph of Quadratic Function with its Main


Characteristics
Before sketching the graph of quadratic function, we need to define some of the following terms:

Symmetry of a Function Y

Symmetry of a function about y-axis 10


y = x2
9
A curve representing the equation f (x, y) = 0 is 8
said to be symmetric about y-axis, if there is no 7
change in the value of y if x is replaced by -x.
6
For example: Consider a function y = f (x) = x2, 5
then; we can construct a graph with following 4

values: 3
2
x –3 –2 –1 0 1 2 3
1
y 9 4 1 0 1 4 9
X' X
O A
The graph of y = x2 is symmetric about y–axis -4 -3 -2 -1 1 2 3 4

and is shown alongside. Y'

Y
Symmetry of a Function about x-axis
5
A curve representing the equation
4
f (x, y) = 0 is said to be symmetric about x-axis if x = y2
3
there is no change in the value of x if y is
2
replaced by –y.
1
A
For example: Consider a function y = x . This X' X
O
can be written as x = y2. -2 -1 1 2 3 4 5 6 7 8 9 10
-2
x 9 4 1 0 1 4 9
-3
y –3 –2 –1 0 1 2 3
-4

The graph of y = x is symmetric about x–axis -5

and is shown alongside. Y'

Zero of a Function
Let f (x) be any function. The values of x at which the function become zero are called the zeros of the
function. Geometrically, the zero of a function are the values of x, for which the graph of the function
touches x-axis.
Example 1. Find the zeros of the function: f (x) = x2 – 7x + 12.
Solution
For the zeros of the function, put f (x) = 0
x2 – 7x + 12 = 0
or, x2 – 4x – 3x + 12 = 0
or, (x – 4) (x – 3) = 0
∴ x = 4, 3.
66 BUSINESS MATHEMATICS I

Main Characteristics for Sketching the graphs of quadratic function y = ax2 + bx + c, (a ≠ 0).
1. Openness: If a > 0, the parabola opens upward and if a < 0 the parabola opens downwards.
b 4ac – b2
2. Vertex = ⎛– 2a‚ 4a ⎞
⎝ ⎠
b
3. Axis of parabola: x = – 2a is the axis of the parabola (axis of symmetry)
2
⎧⎪⎛4ac – b ‚ ∞⎞ if a > 0
⎝ 4a ⎠
4. Domain = (–∞, ∞) and Range = ⎨ 4ac – b 2 .
⎪⎩⎛⎝–∞‚ 4a ⎞⎠ if a < 0
5. Intercepts: For x-intercept, put y = 0 and for y-intercept, put x = 0 in the given function.
Example 2. Sketch the graph of the function f (x) = x2 – 4x + 3 stating its main characteristics.
Solution
Given function is y = x2 – 4x + 3
The characteristics of given function are:
i. The given function y = x2 – 4x + 3 is in the form y = ax2 + bx + c, so it represents a
parabola.
Here, a = 1, b = – 4, c = 3
Since a = 1 > 0, the parabola turns upward.
–b – (– 4)
ii. Vertex: x = 2a = 2 × 1 = 2
When x = 2, y = 22 – 4 × 2 + 3 = –1
∴ Vertex = (2, –1)
iii. Domain = (–∞, ∞)
Range = [–1, ∞)
iv. Axis of the parabola: The line x = 2 is the axis of the parabola.
The parabola is symmetric about x = 2.
v. When x = 0, y = 02 – 4 × 0 + 3 = 3
So, the curve cuts y-axis at (0, 3)
When y = 0, x2 – 4x + 3 = 0
(x – 1) (x – 3) = 0
∴ x = 1, 3
So, the curve cuts x-axis at (1, 0) and (3, 0).
With the above characteristics, the sketch of the graph of given function is as shown
below.
Y
x=2

(0,3)

(1,0)
X' X
O (3,0)

(2,–1)

Y'
Quadratic Functions and Equations and Its Applications UNIT 3 67

Example 3. Sketch the graph of y = –x2 + 4x – 3 indicating its characteristics.


Solution
Given function is y = –x2 + 4x –3
The characteristics of given function are as follows:
i. The given function y = –x2 + 4x – 3 is in the form of y = ax2 + bx + c, it represents a
parabola.
Here, a = –1, b = 4, c = –3
Since a = –1 < 0, the parabola turns downwards.
b 4
ii. Vertex: x = –2a = – 2 × (–1) = 2
When x = 2, y = – 22 + 4 × 2 –3
=– 4 + 8 –3 = 1
∴ Vertex = (2, 1)
iii. Domain = (– ∞, ∞)
Range = (– ∞, 1]
iv. Axis of the parabola: The axis of the parabola is x = 2. The parabola is symmetric
about x = 2.
v. When x = 0, y = –02 + 4 × 0 – 3 = –3. So, the curve cuts y-axis at (0, –3)
vi. When y = 0, –x2 + 4x –3 = 0
or, x2 – 4x + 3 = 0
or, (x – 1) (x –3) = 0
∴ x = 1, 3
So, the curve cuts x-axis at (1, 0) and (3, 0).
With the above characteristics the sketch of the graph of given function is as shown
below.
Y

(2,1)

(3,0)
X' X
O (1,0)

(0,–3)

Y' x=2

3.3 Quadratic Equation


An equation of the form ax2 + bx + c = 0, (a ≠ 0) is called a quadratic equation.
Solution of Quadratic Equation
Consider a quadratic equation
ax2 + bx + c = 0 . . . (i), a ≠ 0
Dividing both the sides by a, we get
b c
x2 + a x + a = 0
68 BUSINESS MATHEMATICS I

b b 2 b 2 c
or, x2 + 2x ⋅ 2a + ⎛2a⎞ – ⎛2a⎞ + a = 0
⎝ ⎠ ⎝ ⎠
2 2
or, ⎛x + b ⎞ = b 2 – c
⎝ 2a⎠ 4a a
2 2
or, ⎛x + b ⎞ = b – 4ac
⎝ 2a⎠ 4a2
2
⎛x + b ⎞ = ⎛± b – 4ac⎞
2 2
or,
⎝ 2a⎠ ⎝ 2a ⎠
b b2 – 4ac
or, x + 2a = ± 2a
b b2 – 4ac
or, x = 2a ± 2a
–b ± b2 – 4ac
∴ x= 2a .

The use of the plus and minus sign gives two roots of the quadratic equation. Hence, the quadratic
equation has two roots. We can solve quadratic equation by different methods. In following examples, we
have solved the quadratic equations by three different methods.
Example 4. Solve the quadratic equation: x2 – 7x + 6 = 0 by (a) Factorization method, (b) Completing
square method (c) Using formula.
Solution
a. Factorization method:
x2 – 7x + 6 = 0
or, x2 – 6x – x + 6 = 0
or, x(x – 6) – 1(x – 6) = 0
or, (x – 1) (x – 6) = 0
Either or,
x–1=0 x–6=0
⇒ x=1 ⇒ x=6
∴ x = 1, 6.
b. Completing square method:
x2 – 7x + 6 = 0
7 7 2 7 2
or, x2 – 2x ⋅ 2 + ⎛2⎞ – ⎛2⎞ + 6 = 0
⎝ ⎠ ⎝ ⎠
2
7 49
⎛x – ⎞ – + 6 = 0
or,
⎝ 2⎠ 4
2
or, ⎛x – 7⎞ = 49 – 6
⎝ 2⎠ 4
2
or, ⎛x – 7⎞ = 49 – 24
⎝ 2⎠ 4
2
or, ⎛x – 7⎞ = 25
⎝ 2⎠ 4
2 2
or, ⎛x – 7 ⎞ = ⎛± 5 ⎞
⎝ 2⎠ ⎝ 2 ⎠
Quadratic Functions and Equations and Its Applications UNIT 3 69

Either or,
7 5 7 5
x–2=2 x–2=2
5 7 5 7
or, x =2+2 or, x = 2 + 2
5+7 –5+7
= 2 = 2
12 2
= 2 =6 =2=1
∴ x = 6, 1.
c. Using formula:
Given quadratic equation is
x2 – 7x + 6 = 0 . . . (i)
Comparing equation (i) with quadratic equation: ax2 + bx + c = 0, we get a = 1, b = –7,
c=6
Using formula;
–b ± b2 – 4ac
x = 2a
–(–7) ± (–7)2 – 4 × 1 × 6
= 2×1
7 ± 49 – 24
= 2
7 ± 25 7 ± 5
= 2 = 2
Take '+' ve sign Take '–' ve sign
7+5 7–5
or, x = 2 or, x = 2
12 2
= 2 =6 =2=1
∴ x = 6, 1.

3.4 Nature of the Roots of Quadratic Equations


−b + b2 − 4ac −b – b2 − 4ac
The roots of the quadratic equation ax2 + bx + c = 0, a ≠ 0 are α = 2a and β = 2a . Here,
the term b2 − 4ac is called the discriminant, which determines the nature of roots of the equation. We
have the following cases:
a. If b2 − 4ac > 0, then the roots are real and distinct. Now, we have following two cases.
i. If b2 − 4ac > 0 and a perfect square, then the roots are rational and distinct for a, b, c rational.
Example: 3x2 + 5x + 2 = 0 has discriminant; 52 − 4 ⋅ 3 ⋅ 2 = 1 > 0 and a perfect square.
Therefore, the roots are real and distinct (rational).
ii. If b2 − 4ac > 0 and not a perfect square, then the roots are irrational and distinct.
Example: 2x2 + 6x + 1 = 0 has discriminant; 36 − 4 ⋅ 2 ⋅ 1 = 28 > 0, not a perfect square.
Therefore, the roots are real (irrational) and distinct.
70 BUSINESS MATHEMATICS I

b. If b2 − 4ac = 0, then the roots are real and equal.


Example: x2 + 4x + 4 = 0 has discriminant; 42 − 4 ⋅ 1 ⋅ 4 = 0. Therefore, the roots are real and equal.
c. If b2 − 4ac < 0, then the roots are imaginary and distinct.
Example: x2 + x + 1 = 0 has discriminant; 12 – 4 ⋅ 1 ⋅ 1 = −3. Therefore, the roots are imaginary and
distinct.
NOTE If a quadratic equation has an irrational root, then other root will be its conjugate. That is, if p + q be one
root, the other root will be p − q.

Example 5. Determine the nature of roots of the equation 4x2 + 8x – 5 = 0.


Solution
Given equation is
4x2 + 8x – 5 = 0 . . . (i)
2
Comparing equation (i) with ax + bx + c = 0, we get
a = 4, b = 8, c = –5
Now
b2 – 4ac = 82 – 4 × 4 × (–5)
= 64 + 80
= 144 which is a perfect square.
Hence, the roots of the given equations are rational, real and unequal.
Example 6. If the equation x2 + 2(k + 2) x + 9k = 0 has equal roots, find k.
Solution
Given equation is
x2 + 2(k + 2) x + 9k = 0 . . . (i)
Comparing equation (i) with ax2 + bx + c = 0, we get
a = 1, b = 2(k + 2) and c = 9k
If the equation (i) has be equal roots then b2 – 4ac = 0
∴ {2(k + 2)}2 − 4⋅1⋅9k = 0
or, 4(k2 + 4k + 4) − 36k = 0
or, k2 − 5k + 4 = 0
or, k2 − 4k − k + 4 = 0
or, k(k − 4) − 1(k − 4) = 0
or, (k − 4) (k − 1) = 0
∴ k = 1, 4.

Relation between Roots and Coefficients


Let α and β be the roots of the quadratic equation
ax2 + bx + c = 0 then
− b + b2 − 4ac − b − b2 − 4ac
α= 2a and β = 2a .
Quadratic Functions and Equations and Its Applications UNIT 3 71

Now Again
2 2
− b + b − 4ac − b − b − 4ac − b + b2 − 4ac − b − b2 − 4ac
α+β = 2a + 2a αβ = 2a ⋅ 2a
b
= − 2a +
2
b − 4ac b
− 2a −
2
b − 4ac ⎛− b b − 4ac⎞ ⎛−b
2
b2 − 4ac ⎞
2a 2a = 2a +
⎝ 2a ⎠ ⋅ ⎝2a − 2a ⎠
2 2
2b −b ⎛ b2 − 4ac ⎞
= − 2a = ⎛ 2a ⎞ −
⎝ ⎠ ⎝ 2a ⎠
b b2 b2 − 4ac c
= −a. = 4a2 − 4a2 = a .

b coefficient of x c constant term


Hence, sum of the roots = − a = − coefficient of x2 and product of the roots = a = coefficient of x2 .

Formulation of Quadratic Equation


Let α and β be the roots of a quadratic equation
ax2 + bx + c = 0, (a ≠ 0). . . . (1)
b c
Then, α + β = – a and α ⋅ β = a .
Equation (1) can be written as
b c
x2 + a x + a = 0 , (∵ a ≠ 0)

−b
x2 − ⎛ a ⎞ x + a = 0
c
or,
⎝ ⎠
i.e., x2 − (α + β) x + αβ = 0
Hence, any quadratic equation can be expressed as
x2 − (sum of roots) x + (product of roots) = 0.
Example 7. Find the quadratic equation whose roots are 3, 2.
Solution
Sum of roots = 3 + 2 = 5
Product of roots = (3) (2) = 6
The quadratic equation is
x2 – (sum of roots) x + product of roots = 0
or, x2 – 5x + 6 = 0
∴ x2 – 5x + 6 = 0.

3.5 Quadratic Inequalities


The inequalities involving quadratic functions are called quadratic inequalities.
The solution of quadratic inequalities can be written in inequality notation or interval notation. In general,
we apply the following steps to solve quadratic inequalities.
1. Write all the terms to one side of the inequality sign making other side zero.
2. Change the quadratic inequality to an equation and solve it.
3. Put the roots (solutions) of the equations on a number line.
72 BUSINESS MATHEMATICS I

4. Form a table showing a sign line to show where the expression in the inequality is positive or
negative.
5. Find the solution and write it in inequality or interval notation.
Example 8. Solve x2 – 9 < 0.
Solution
x2 – 9 < 0
⇒ x2 < 9
⇒ | x |2 < | 3 |2 [∵ x2 = | x |2]
⇒ |x|<3
⇒ –3 < x < 3 [∵ | x | < a ⇔ – a < x < a]
Alternative method
The corresponding equation of given inequality is –∞ –3 3 ∞
x2 – 9 = 0
or, (x + 3) (x – 3) = 0
∴ x = –3, 3
Now, these two points divide the real line into 3 sub intervals (–∞, –3), (–3, 3) and (3, ∞).
Sign of
Intervals
(x – 3) (x + 3) (x – 3) (x + 3)
(–∞, –3) –ve – ve + ve
(–3, 3) + ve – ve – ve
(3, ∞) + ve + ve + ve
Thus, the possible interval is (–3, 3).
We plot the graph with open circles on –3 and 3 as follows.

. . . -5 -4 -3 -2 -1 0 1 2 3 4 5 6 7 8 ...
2
Example 9. Solve: x – 2x – 3 > 0.
Solution
The corresponding equation of given inequality is
x2 – 2x – 3 = 0
or, x2 – 3x + x – 3 = 0
or, x(x – 3) + 1(x – 3) = 0
or, (x + 1) (x – 3) = 0 –∞ –1 3 ∞
∴ x = –1, 3
Now, these two points divide the real line into 3 sub-intervals (–∞, –1), (–1, 3) and (3, ∞).
Sign of
Intervals
(x + 1) (x – 3) (x + 1) (x – 3)
(–∞, –1) –ve – ve + ve
(–1, 3) + ve – ve – ve
(3, ∞) + ve + ve + ve
∴ The possible interval is (–∞, –1) ∪ (3, ∞).
The graph of the solution set is
x < –1 x>3

. . . -5 -4 -3 -2 -1 0 1 2 3 4 5 6 7 8 ...
Quadratic Functions and Equations and Its Applications UNIT 3 73

Example 10. Solve the inequality: 6 + 5x – x2 ≥ 0.


Solution
The corresponding equation of given inequality is
6 + 5x – x2 = 0
–∞ –1 6 ∞
or, (6 – x) (x + 1 ) = 0
∴ x = –1, x = 6
These two points divide the whole real line into 3 sub-intervals (–∞, –1), (–1, 6) and (6, ∞).
Sign of
Intervals
(6 – x) (x + 1) (6 –x) ( x+ 1)
(–∞, –1) + ve – ve – ve
(–1, 6) + ve + ve + ve
(6, ∞) – ve + ve – ve
Also, at x = –1, and x = 6, 6 + 5x – x2 = 0
The possible interval is (–1, 6) ∪ {–1, 6} = [–1, 6].
The graph of solution set is
-1 ≤ x ≤ 6

. . . -5 -4 -3 -2 -1 0 1 2 3 4 5 6 7 8 ...

 Exercise 3.1
1. Find the zero(s) of the following functions.
a. f (x) = x + 9 b. f (x) = x2 – 11x + 28 c. f (x) = 4x2 – 11x + 15.
2. Solve the following quadratic equations.
4
a. x2 – 6x + 5 = 0 b. 2x2 – 5x + 3 = 0 c. x+x=5
5 3
d. x + 4 – x – 2 = 4.
3. Discuss the nature of the roots of the following equations.
a. x2 − 4x + 4 = 0 b. x2 + 2x + 3 = 0 c. x2 + 5x + 6 = 0
d. 2x2 + 5x + 1 = 0.
4. Find the quadratic equation whose roots are or one of the roots is
a. –3, 4 b. 2 + 3 .
5. a. For what values of p will the equation 5x2 – px + 45 = 0 have equal roots?
b. For what values of k will the equation 9x2 + kx + 1 = 0 have equal roots?
6. Sketch the graph of following functions with characteristics.
a. y = x2 + 4x + 3 b. y = x2 + x + 2 c. y = x2 – 6x + 5.
2 2
d. y = x – x e. y = –x + 4x – 3
7. Solve the following quadratic inequalities.
a. 2x2 – 7x + 6 < 0 b. 3x2 – 8x + 5 ≤ 0 c. –x2 + 4x – 3 ≤ 0
2
d. x + 6x + 8 > 0.

ANSWERS
5
1. (a) – 9 (b) 4, 7 (c) 3, 4
74 BUSINESS MATHEMATICS I

3 5
2. (a) 5, 1 (b) 2 , 1 (c) 1, 4 (d) 1, – 2

3. (a) Real and Equal (b) Imaginary and dIstinct (c) Rational and Distinct (d) Irrational and Distinct
4. (a) x2 + x – 20 = 0 (b) x2 – 4x + 1 = 0
5. (a) ± 30 (b) ± 6
6. (a) (b)
Y Y

y = x2 + 4x + 3
y = x2 + x + 2
(0, 3)

(0, 2)

X'
(-3, 0)
(-1, 0)
O
X
X'
( )1 7
–2‚ 4
X
O
(-2, -1)
1
Y' x = – 2 Y'
x = –2
(c) Y (d)
Y
2
y = x – 6x + 5

(12 ‚ 14)
(0, 0) (1,0)
X' X
O
(1, 0) (5, 0)
X' X
O

1
x=2

Y'
(3, -4)

x=3
Y'
(e) Y

(2, 1)

(1, 0) (3, 0)
X' X
O

y = –x2 + 4x – 3
(0, -3)

x=2
Y'

7. (a) (32‚ 2) (b) [1‚ 53] (c) (–∞, 1] ∪ [3, ∞) (d) (– ∞, – 4) ∪ (–2, ∞)
Quadratic Functions and Equations and Its Applications UNIT 3 75

Application of Quadratic Functions and


Equations
3.6 Supply and Demand Analysis
In previous chapter, linear functions are used for demand and supply analysis. In this section, we use
quadratic functions to do the demand and supply analysis. We discuss this with the help of following
examples.
Example 1. The demand and supply functions for a market are given by Pd = 125 – Q2 and Ps = 0.2Q2 + 5
respectively. Find the equilibrium price and quantity.
Solution
Given Pd = 125 – Q2 and Ps = 0.2Q2 + 5.
For equilibrium condition, we have
P d = Ps
or, 125 – Q2 = 0.2Q2 + 5
or, 125 – 5 = 0.2Q2 + Q2
or, 1.2Q2 = 120
120
or, Q2 = 1.2 = 100
∴ Q = 10
Substituting the value of Q in Pd or Ps, P = 125 – 102 = 25.
∴ Equilibrium price = 25, and equilibrium quantity = 10.

3.7 Break-Even Analysis


In this section, we discuss break-even analysis with the help of quadratic functions. The concept is same
as that of the linear function which was discussed in previous chapter. The point at which total cost (TC)
and total revenue (TR) are equal is called a break even point.
Example 2. If the demand function for a commodity is P = 15 – Q and the cost function is TC = Q2 –
3Q – 20, where Q is output, find the break-even point.
Solution
Total cost function (TC) = Q2 – 3Q – 20
Total revenue function (TR) = Demand function × Quantity
= (15 – Q) × Q
= 15Q – Q2
For break-even point,
TC = TR
or, Q2 – 3Q – 20 = 15Q – Q2
or, 2Q2 – 3Q – 15Q – 20 = 0
or, 2Q2 – 18Q – 20 = 0
76 BUSINESS MATHEMATICS I

or, Q-2 – 9Q – 10 = 0
or, Q2 – 10Q + Q – 10 = 0
or, Q(Q – 10) + 1(Q – 10) = 0
or, (Q + 1) (Q – 10) = 0
Either Q + 1 = 0 ⇒ Q = – 1 (not possible)
or, Q – 10 = 0 ⇒ Q = 10.
∴ Q = 10 is the break-even point.
Example 3. The demand function for a good is P = 20 – 2Q. Fixed costs are Rs. 40 and each unit
produced costs of an additional Rs. 2.
a. Write down the expressions for TR and TC in terms of Q.
b. Find break-even point(s).
Solution
a. Total revenue function (TR) = Demand function × Quantity
= (20 – 2Q) × Q
= 20Q – 2Q2
Total cost function (TC) = Fixed cost + Variable cost
= 40 + 2 × Q
= 2Q + 40
b. For break-even point(s),
TR = TC
20Q – 2Q2 = 2Q + 40
or, 0 = 2Q2 – 20Q + 2Q + 40
or, 2Q2 – 18Q + 40 = 0
or, Q2 – 9Q + 20 = 0
or, Q2 – 5Q – 4Q + 20 = 0
or, Q(Q – 5) – 4(Q – 5) = 0
or, (Q – 4) (Q – 5) = 0
∴ Q = 4, 5 are the break-even points.

3.8 Optimization
A businessperson wants to minimize costs and maximize profits. In this section, we discuss such
maximization and minimization problems. In solving such practical problems, we have to convert the
word problem into a mathematical optimization problem by setting up the function that is to be
maximized or minimized.
For the quadratic function y = ax2 + bx + c, (a ≠ 0), two cases arise which are discussed below:
b 4ac – b2
1. If a > 0, then the parabola opens upwards and the vertex ⎛–2a‚ 4a ⎞ is the minimum point. That
⎝ ⎠
4ac – b2 b
is, minimum value of the function = 4a at x = – 2a . In this case, maximum value of y does not
exist.
Quadratic Functions and Equations and Its Applications UNIT 3 77

b 4ac – b2
2. If a < 0, then the parabola opens downward and the vertex ⎛–2a‚ 4a ⎞ is the maximum point.
⎝ ⎠
4ac – b2 b
That is, maximum value of the function = 4a at x = – 2a . In this case, minimum value of y
does not exist.
Example 4. The demand function of a good is given by P = 100 – 2Q. Find total revenue function (TR)
and sketch it. Find the maximum value of TR.
Solution
Total revenue function (TR) = P × Q
= (100 – 2Q) × Q
= 100Q – 2Q2
This is a quadratic function.
The main characteristics are:
i. Since the coefficient of Q2 is negative, the parabola opens downward.
ii. When Q = 0, TR = 0
When TR = 0, 100Q – 2Q2 = 0
or, Q(100 – 2Q) = 0
∴ Q = 0, 50.
iii. By symmetry, the parabola attains its maximum halfway between 0 and 50. Thus, the
parabola attains its maximum at Q = 25.
The corresponding revenue function
TR = 100(25) – 2(25)2
= 1250
Thus, the maximum value of TR is 1250 at Q = 25. This is shown in the following figure.
Y

(25, 1250)

TR
TR = 100Q – 2Q2

X
O 50

Quantity (Q)
Example 5. If fixed costs are 4, variable costs per unit are 1 and the demand function is P = 10 – 2Q.
Find the maximum profit.
Solution
Total cost function (TC) = Fixed costs + Variable costs
=4+Q
Total revenue function (TR) = P × Q
= (10 – 2Q) × Q
= 10Q – 2Q2
78 BUSINESS MATHEMATICS I

Profit function (π) = TR – TC


= (10Q – 2Q2) – (4 + Q)
= – 2Q2 + 9Q – 4
Here, a = – 2, b = 9, c = – 4
Since a = – 2 < 0, the parabola representing profit function opens downward. Hence, it
attains maximum at vertex.
–b 4ac – b2
Vertex = ⎛2a‚ 4a ⎞
⎝ ⎠
–9 4 × (– 2) × (– 4) – 92⎞
= ⎛2 × (– 2)‚
⎝ 4 × (– 2) ⎠
= (2.25, 6.125)
∴ Maximum profit = 6.125 at Q = 2.25.

 Exercise 3.2
1. The demand and supply for a market are given by Pd = Q2 – 6Q + 20 and Ps = Q2 + 2Q + 4
respectively. Find the equilibrium price and quantity.
2. The demand and supply functions are given by Pd = 155 – Q2 and Ps = 0.5Q2 + 5 respectively. Find
the equilibrium price and quantity.
3. The demand function is given by P = 25 – 2Q and total cost function (TC) = 32 + 5Q. Find the
values of Q for which the firm breaks even.
4. The demand function of a firm is given by P + 2Q = 50 and total cost function is Q2 + 3Q + 30. Find
the break-even points and profit function.
5. If the total cost function of a firm is given by C = Q2 – 6Q + 25 then find the value of minimum cost.
6. The demand function for a firm's product is Q = 150,000 – 75P where Q equals the number of units
demanded and P equals the price in dollars. Determine the price which should be charged to
maximize the total revenue. What is the maximum value of total revenue?
7. Given the demand function P = 20 – Q and the total cost function C = Q2 + 8Q + 2, find the value of
maximum profit.
8. If the revenue function is R = Q – 3Q2 and the cost function C = Q2 – 2Q, where Q is the number of
units. Find the value of Q at which the profit is maximum. Also find the maximum profit.
9. A manufacturer of dishwashers finds the demand function p + 3x = 600, p is the price in dollars; x is
the number of dishwashers produced and sold. The cost of production is C(x) = 400 + 150x + 0.5x2.
a. Find the revenue function
b. Find the profit function
c. Find the production level for which profit is maximum.

ANSWERS
2
1. 12, 2 2. 55, 10 3. Q = 2, 8 4. Q = 3 , 15; –3Q2 + 47Q – 30

5. Min. cost = 16 at Q = 3. 6. $1000, $750,00,000 7. Max. profit = 16 at Q = 3


9 3
8. Maximum profit is 16 at Q = 8

9. (a) 600x – 3x2 (b) 450x – 3.5x2 – 400 (c) 64.28


Quadratic Functions and Equations and Its Applications UNIT 3 79

3.9 Quadratic Functions and Equations using MS Excel


1. Sketch the graph: f (x) = y = x2

Note: Use of Solver is given in LPP.


2. Given the demand function P = 20 – Q and the total cost function C = Q2 + 8Q + 2, find the value of
maximum profit.
Here, Profit function, π =R–C
= (20 – Q) Q – (Q2 + 8Q + 2)
= –2Q2 + 12Q – 2
∴ π = –2Q2 + 12Q – 2
80 BUSINESS MATHEMATICS I

Therefore, maximum value of profit = 16 at Q = 3.



You might also like